102 Final

Lakukan tugas rumah & ujian kamu dengan baik sekarang menggunakan Quizwiz!

A nurse is a preceptor for a nurse who just graduated from nursing school. When caring for a patient, the new graduate nurse begins to explain to the patient the purpose of completing a physical assessment. Which statement made by the new graduate nurse requires the preceptor to intervene? A. "I will use the information from my assessment to figure out if your antihypertensive medication is working effectively." B. "Nursing assessment data are used only to provide information about the effectiveness of your medical care." C. "Nurses use data from their patient's physical assessment to determine a patient's educational needs." D. "Information gained from physical assessment helps nurses better understand their patients' emotional needs."

B. "Nursing assessment data are used only to provide information about the effectiveness of your medical care." Nursing assessment data are used to evaluate the effectiveness of all aspects of a patient's care, not just the patient's medical care. Assessment data help to evaluate the effectiveness of medications and to determine a patient's health care needs, including the need for patient education. Nurses also use assessment data to identify patients' psychosocial and cultural needs.

The nurse is assessing an adult patient's patellar reflex. Which finding will the nurse record as normal? A. 1+ B. 2+ C. 3+ D. 4+

B. 2+ Grade reflexes as follows: 0: No response; 1+: Sluggish or diminished; 2+: Active or expected response; 3+: More brisk than expected, slightly hyperactive; and 4+: Brisk and hyperactive with intermittent or transient clonus.

A nurse is performing a mental status examination and asks an adult patient what the statement "Don't cry over spilled milk" means. Which area is the nurse assessing? A. Long-term memory B. Abstract thinking C. Recent memory D. Knowledge

B. Abstract thinking For an individual to explain common phrases such as "A stitch in time saves nine" or "Don't cry over spilled milk" requires a higher level of intellectual function or abstract thinking. Knowledge-based assessment is factual. Assess knowledge by asking how much the patient knows about the illness or the reason for seeking health care. To assess past (long-term) memory, ask the patient to recall the maiden name of the patient's mother, a birthday, or a special date in history. It is best to ask open-ended questions rather than simple yes/no questions. Patients demonstrate immediate recall (recent memory) by repeating a series of numbers in the order in which they are presented or in reverse order.

A teen female patient reports intermittent abdominal pain for 12 hours. No dysuria is present. Which action will the nurse take when performing an abdominal assessment? A. Assess the area that is most tender first. B. Ask the patient about the color of her stools. C. Recommend that the patient take more laxatives. D. Avoid sexual references such as possible pregnancy.

B. Ask the patient about the color of her stools. Abdominal pain can be related to bowels. If stools are black or tarry (melena), this may indicate gastrointestinal alteration. The nurse should caution patients about the dangers of excessive use of laxatives or enemas. There is not enough information about the abdominal pain to recommend laxatives. Determine if the patient is pregnant, and note her last menstrual period. Pregnancy causes changes in abdominal shape and contour. Assess painful areas last to minimize discomfort and anxiety.

A teen patient is tearful and reports locating lumps in her breasts. Other history obtained is that she is currently menstruating. Physical examination reveals soft and movable cysts in both breasts that are painful to palpation. The nurse also notes that the patient's nipples are erect, but the areola is wrinkled. Which action will the nurse take after talking with the health care provider? A. Reassure patient that her symptoms are normal. B. Discuss the possibility of fibrocystic disease as the probable cause. C. Consult a breast surgeon because of the abnormal nipples and areola. D. Tell the patient that the symptoms may get worse when her period ends.

B. Discuss the possibility of fibrocystic disease as the probable cause. A common benign condition of the breast is benign (fibrocystic) breast disease. This patient has symptoms of fibrocystic disease, which include bilateral lumpy, painful breasts sometimes accompanied by nipple discharge. Symptoms are more apparent during the menstrual period. When palpated, the cysts (lumps) are soft, well differentiated, and movable. Deep cysts feel hard. Although a common condition, benign breast disease is not normal; therefore, the nurse does not tell the patient that this is a normal finding. During examination of the nipples and areolae, the nipple sometimes becomes erect with wrinkling of the areola. Therefore, consulting a breast surgeon to treat her nipples and areolae is not appropriate.

While assessing the skin of an 82-year-old patient, a nurse discovers nonpainful, ruby red papules on the patient's trunk. What is the nurse's next action? A. Explain that the patient has basal cell carcinoma and should watch for spread. B. Document cherry angiomas as a normal older adult skin finding. C. Tell the patient that this is a benign squamous cell carcinoma. D. Record the presence of petechiae.

B. Document cherry angiomas as a normal older adult skin finding. The skin is normally free of lesions, except for common freckles or age-related changes such as skin tags, senile keratosis (thickening of skin), cherry angiomas (ruby red papules), and atrophic warts. Basal cell carcinoma is most common in sun-exposed areas and frequently occurs in a background of sun-damaged skin; it almost never spreads to other parts of the body. Squamous cell carcinoma is more serious than basal cell and develops on the outer layers of sun-exposed skin; these cells may travel to lymph nodes and throughout the body. Report abnormal lesions to the health care provider for further examination. Petechiae are nonblanching, pinpoint-size, red or purple spots on the skin caused by small hemorrhages in the skin layers.

An older-adult patient is taking aminoglycoside for a severe infection. Which assessment is the priority? A. Eyes B. Ears C. Skin D. Reflexes

B. Ears Older adults are especially at risk for hearing loss caused by ototoxicity (injury to auditory nerve) resulting from high maintenance doses of antibiotics (e.g., aminoglycosides). While eyes and skin are important, they are not the priority. Reflexes are expected to be diminished in older adults.

An older-adult patient is being seen for chronic entropion. Which condition will the nurse assess for in this patient? A. Ptosis B. Infection C. Borborygmi D. Exophthalmos

B. Infection The diagnosis of entropion can lead to lashes of the lids irritating the conjunctiva and cornea. Irritation can lead to infection. Exophthalmos is a bulging of the eyes and usually indicates hyperthyroidism. An abnormal drooping of the lid over the pupil is called ptosis. In the older adult, ptosis results from a loss of elasticity that accompanies aging. Hyperactive sounds are loud, "growling" sounds called borborygmi, which indicate increased GI motility.

The nurse is examining a female with vaginal discharge. Which position will the nurse place the patient for proper examination? A. Sitting B. Lithotomy C. Knee-chest D. Dorsal recumbent

B. Lithotomy Lithotomy is the position for examination of female genitalia. The lithotomy position provides for the maximum exposure of genitalia and allows the insertion of a vaginal speculum. Sitting does not allow adequate access for speculum insertion and is better used to visualize upper body parts. Dorsal recumbent is used to examine the head and neck, anterior thorax and lungs, breasts, axillae, heart, and abdomen. Knee-chest provides maximal exposure of the rectal area but is embarrassing and uncomfortable.

A nurse observes smoke coming from under the door of a patient's room. What should the nurse do after rescuing the patients in the room? A) Extinguish the fire. B) Pull the fire alarm. C) Close all open doors on the unit. D) Aim at the base of the fire

B: RACE (rescue, alarm, contain, extinguish)

A nurse is a preceptor for a nurse who just graduated from nursing school. When caring for a patient, the new graduate nurse begins to explain to the patient the purpose of completing a physical assessment. Which statement made by the new graduate nurse requires the preceptor to intervene? A) "I will use the information from my assessment to figure out if your antihypertensive medication is working effectively." B) "Nursing assessment data are used only to provide information about the effectiveness of your medical care." C) "Nurses use data from their patient's physical assessment to determine a patient's educational needs." D) "Information gained from physical assessment helps nurses better understand their patients' emotional needs."

B: negatively worded question, only B requires the nurse to intervene, we use assessment data to determine the baseline on the status of our patient and to see what is wrong

During a routine physical examination of a 70-year-old patient, a blowing sound is auscultated over the carotid artery. Which assessment finding will the nurse report to the health care provider?

Bruit

A female adult patient presents to the clinic with reports of a white discharge and itching in the vaginal area. A nurse is taking a health history. Which question is the priority? a."When was the last time you visited your primary health care provider?" b."Has this condition affected your eating habits in any way?" c."What medications are you currently taking?" d."Are you able to sleep at night?"

C

The nurse is caring for a group of medical-surgical patients. Which patient is most at risk for developing an infection? a.A patient who is in observation for chest pain b.A patient who has been admitted with dehydration c.A patient who is recovering from a right total hip surgery d.A patient who has been admitted for stabilization of heart problems

C

The nurse is caring for a patient in an intensive care unit who needs a bath. Which priorityaction will the nurse take to decrease the potential for a health care-associated infection? a.Use local anesthetic on reddened areas. b.Use nonallergenic tape on dressings. c.Use a chlorhexidine wash. d.Use filtered water.

C

The nurse is caring for a patient in labor and delivery. When near completing an assessment of the patient's cervix, the electronic infusion device being used on the intravenous (IV) infusion alarms. Which sequence of actions is most appropriate for the nurse to take? a.Complete the assessment, remove gloves, and silence the alarm. b.Discontinue the assessment, silence the alarm, and assess the intravenous site. c.Complete the assessment, remove gloves, wash hands, and assess the intravenous infusion. d.Discontinue the assessment, remove gloves, use hand gel, and assess the intravenous infusion.

C

The nurse is caring for a patient in the endoscopy area. The nurse observes the technician performing these tasks. Which observation will require the nurse to intervene? a.Washing hands after removing gloves b.Disinfecting endoscopes in the workroom c.Removing gloves to transfer the endoscope d.Placing the endoscope in a container for transfer

C

The nurse is caring for a patient who has just delivered a neonate. The nurse is checking the patient for excessive vaginal drainage. Which precaution will the nurse use? a.Contact b.Droplet c.Standard d.Protective environment

C

The nurse is caring for a patient who is susceptible to infection. Which instruction will the nurse include in an educational session to decrease the risk of infection? a.Teaching the patient about fall prevention b.Teaching the patient to take a temperature c.Teaching the patient to select nutritious foods d.Teaching the patient about the effects of alcohol

C

The nurse is caring for a patient with an incision. Which actions will best indicate an understanding of medical and surgical asepsis for a sterile dressing change? a.Donning clean goggles, gown, and gloves to dress the wound b.Donning sterile gown and gloves to remove the wound dressing c.Utilizing clean gloves to remove the dressing and sterile supplies for the new dressing d.Utilizing clean gloves to remove the dressing and clean supplies for the new dressing

C

The nurse is caring for a patient with leukemia and is preparing to provide fluids through a vascular access (IV) device. Which nursing intervention is a priority in this procedure? a.Review the procedure with the patient. b.Position the patient comfortably. c.Maintain surgical aseptic technique. d.Gather available supplies.

C

The nurse is caring for an adult patient in the clinic who has been evacuated and is a victim of flooding. The nurse teaches the patient about rest, exercise, and eating properly and how to utilize deep breathing and visualization. What is the primary rationale for the nurse's actions related to the teaching? a.Topics taught are standard information taught during health care visits. b.The patient requested this information to teach the extended family members. c.Stress for long periods of time can lead to exhaustion and decreased resistance to infection. d.These techniques will help the patient manage the pain and loss of personal belongings.

C

The patient and the nurse are discussing Rickettsia rickettsii—Rocky Mountain spotted fever. Which patient statement to the nurse indicates understanding regarding the mode of transmission for this disease? a."When camping, I will use sunscreen." b."When camping, I will drink bottled water." c."When camping, I will wear insect repellent." d."When camping, I will wash my hands with hand gel."

C

The nurse is caring for a patient with a spinal cord injury and notices that the patient's hips have a tendency to rotate externally when the patient is supine. Which device will the nurse use to help prevent injury secondary to this rotation? A) Hand rolls B) A trapeze bar C) A trochanter roll D) Hand-wrist splints

C prevent external rotation (hand rolls & hand wrist splints prevent contractors, trapeze bar helps prevent pressure ulcers)

The nurse considers several new female patients to receive additional teaching on the need for more frequent Pap test and gynecological examinations. Which assessment findings reveal the patient at highest risk for cervical cancer and having the greatest need for patient education? A. 13 years old, nonsmoker, not sexually active B. 15 years old, social smoker, celibate C. 22 years old, smokes 1 pack of cigarettes per day, has multiple sexual partners D. 50 years old, stopped smoking 30 years ago, has history of multiple pregnancies

C. 22 years old, smokes 1 pack of cigarettes per day, has multiple sexual partners Females considered to be at higher risk include those who smoke, have multiple sex partners, and have a history of sexually transmitted infections. Of all the assessment findings listed, the 22-year-old smoker with multiple sexual partners has the greatest number of risk factors for cervical cancer. The other patients are at lower risk: not sexually active, celibate, and do not smoke.

The paramedics transport an adult involved in a motor vehicle accident to the emergency department. On physical examination, the patient's level of consciousness is reported as opening eyes to pain and responding with inappropriate words and flexion withdrawal to painful stimuli. Which value will the nurse report for the patient's Glasgow Coma Scale score? A. 5 B. 7 C. 9 D. 11

C. 9 According to the guidelines of the Glasgow Coma Scale, the patient has a score of 9. Opening eyes to pain is 2 points; inappropriate word use is 3 points; and flexion withdrawal is 4 points. The total for this patient is 2 + 3 + 4 = 9.

An advanced practice nurse is preparing to assess the external genitalia of a 25-year-old American woman of Chinese descent. Which action will the nurse do first? A. Place the patient in the lithotomy position. B. Drape the patient to enhance patient comfort. C. Assess the patient's feelings about the examination. D. Ask the patient if she would like her mother to be present in the room.

C. Assess the patient's feelings about the examination. Patients who are Chinese American often believe that examination of the external genitalia is offensive. Before proceeding with the examination, the nurse first determines how the patient feels about the procedure and explains the procedure to answer any questions and to help the patient feel comfortable with the assessment. Once the patient is ready to have her external genitalia examined, the nurse places the patient in the lithotomy position and drapes the patient appropriately. Typically, nurses ask adolescents if they want a parent present during the examination. The patient in this question is 25 years old; asking if she would like her mother to be present is inappropriate.

During a sexually transmitted illness presentation to high-school students, the nurse recommends the human papillomavirus (HPV) vaccine series. Which condition is the nurse trying to prevent? A. Breast cancer B. Ovarian cancer C. Cervical cancer D. Testicular cancer

C. Cervical cancer Human papillomavirus (HPV) infection increases the person's risk for cervical cancer. HPV vaccine is recommended for females aged 11 to 12 years but can be given to females ages 12 through 26; males can also receive the vaccine. HPV is not a risk factor for breast, ovarian, and testicular cancer.

The nurse completed assessments on several patients. Which assessment finding will the nurse record as normal? A. Pulse strength 3 B. 1+ pitting edema C. Constricting pupils when directly illuminated D. Hyperactive bowel sounds in all four quadrants

C. Constricting pupils when directly illuminated A normal finding is pupils constricting when directly illuminated with a penlight. A pulse strength of 3 indicates a full or increased pulse; 2 is normal. 1+ pitting edema is abnormal; there should be no edema for a normal finding. Hyperactive bowel sounds are abnormal and indicate increased GI motility; normal bowel sounds are active.

The patient has had a stroke that has affected the ability to speak. The patient becomes extremely frustrated when trying to speak. The patient responds correctly to questions and instructions but cannot form words coherently. Which type of aphasia is the patient experiencing? A. Sensory B. Receptive C. Expressive D. Combination

C. Expressive The two types of aphasias are sensory (or receptive) and motor (or expressive). The patient cannot form words coherently, indicating expressive or motor aphasia is present. The patient responds correctly to questions and instructions, indicating receptive or sensory aphasia is not present. Patients sometimes suffer a combination of receptive and expressive aphasia, but this is not the case here.

On admission, a patient weighs 250 pounds. The weight is recorded as 256 pounds on the second inpatient day. Which condition will the nurse assess for in this patient? A. Anorexia B. Weight loss C. Fluid retention D. Increased nutritional intake

C. Fluid retention This patient has gained 6 pounds in a 24-hour period. A weight gain of 5 pounds (2.3 kg) or more in a day indicates fluid retention problems, not nutritional intake. A weight loss is considered significant if the patient has lost more than 5% of body weight in a month or 10% in 6 months. A downward trend may indicate a reduction in nutritional reserves that may be caused by decreased intake such as anorexia.

The nurse is assessing skin turgor. Which technique will the nurse use? A. Press lightly on the forearm. B. Press lightly on the fingertips. C. Grasp a fold of skin on the sternal area. D. Grasp a fold of skin on the back of the hand.

C. Grasp a fold of skin on the sternal area. To assess skin turgor, grasp a fold of skin on the back of the forearm or sternal area with the fingertips and release. Since the skin on the back of the hand is normally loose and thin, turgor is not reliably assessed at that site. Pressing lightly on the forearm can be used to assess for pitting edema or pain or sense of touch. Pressing lightly on the fingertips and observing nail color is assessing capillary refill.

During a school physical examination, the nurse reviews the patient's current medical history. The nurse discovers the patient has allergies. Which assessment finding is consistent with allergies? A. Clubbing B. Yellow discharge C. Pale nasal mucosa D. Puffiness of nasal mucosa

C. Pale nasal mucosa Pale nasal mucosa with clear discharge indicates allergy. Clubbing is due to insufficient oxygenation at the periphery resulting from conditions such as chronic emphysema and congenital heart disease; it is noted in the nails. A sinus infection results in yellowish or greenish discharge. Habitual use of intranasal cocaine and opioids causes puffiness and increased vascularity of the nasal mucosa.

The patient presents to the clinic with dysuria and hematuria. How does the nurse proceed to assess for kidney inflammation? A. Uses deep palpation posteriorly. B. Lightly palpates each abdominal quadrant. C. Percusses posteriorly the costovertebral angle at the scapular line. D. Inspects abdomen for abnormal movement or shadows using indirect lighting.

C. Percusses posteriorly the costovertebral angle at the scapular line. With the patient sitting or standing erect, use direct or indirect percussion to assess for kidney inflammation. With the ulnar surface of the partially closed fist, percuss posteriorly the costovertebral angle at the scapular line. If the kidneys are inflamed, the patient feels tenderness during percussion. Use a systematic palpation approach for each quadrant of the abdomen to assess for muscular resistance, distention, abdominal tenderness, and superficial organs or masses. Light palpation would not detect kidney tenderness because the kidneys sit deep within the abdominal cavity. Posteriorly, the lower ribs and heavy back muscles protect the kidneys, so they cannot be palpated. Kidney inflammation will not cause abdominal movement. However, to inspect the abdomen for abnormal movement or shadows, the nurse should stand on the patient's right side and inspect from above the abdomen using direct light over the abdomen.

A nurse is preparing to perform a complete physical examination on a weak, older-adult patient with bilateral basilar pneumonia. Which position will the nurse use? A. Prone B. Sims' C. Supine D. Lateral recumbent

C. Supine Supine is the most normally relaxed position. If the patient becomes short of breath easily, raise the head of the bed. Supine position would be easiest for a weak, older-adult person during the examination. Lateral recumbent and prone positions cause respiratory difficulty for any patient with respiratory difficulties. Sims' position is used for assessment of the rectum and the vagina.

The nurse is assessing the tympanic membranes of an infant. Which action by the nurse demonstrates proper technique? A. Pulls the auricle upward and backward. B. Holds handle of the otoscope between the thumb and little finger. C. Uses an inverted otoscope grip while pulling the auricle downward and back. D. Places the handle of the otoscope between the thumb and index finger while pulling the auricle upward.

C. Uses an inverted otoscope grip while pulling the auricle downward and back. Using the inverted otoscope grip while pulling the auricle downward and back is a common approach with infant/child examinations because it prevents accidental movement of the otoscope deeper into the ear canal, as could occur with an unexpected pediatric reaction to the ear examination. The other techniques could result in injury to the infant's tympanic membrane. Insert the scope while pulling the auricle upward and backward in the adult and older child. Hold the handle of the otoscope in the space between the thumb and index finger, supported on the middle finger.

A nurse is auscultating different areas on an adult patient. Which technique should the nurse use during an assessment? A. Uses the bell to listen for lung sounds B. Uses the diaphragm to listen for bruits C. Uses the diaphragm to listen for bowel sounds D. Uses the bell to listen for high-pitched murmurs

C. Uses the diaphragm to listen for bowel sounds The bell is best for hearing low-pitched sounds such as vascular (bruits) and certain heart sounds (low-pitched murmurs), and the diaphragm is best for listening to high-pitched sounds such as bowel and lung sounds and high-pitched murmurs.

A nurse is preparing to reposition a patient. Which task can the nurse delegate to the nursing assistive personnel? A) Determining the level of comfort B) Identifying immobility hazards C) Changing the patient's position D) Assessing circulation

C: RNs can't delegate assessment or teaching (determining and identifying are other terms for assessment)

A nurse is preparing a care plan for a patient who is immobile. Which psychosocial aspect will the nurse consider? A) Loss of bone mass B) Loss of strength C) Loss of hope D) Loss of weight

C: psychosocial (the rest are physical)

During a sexually transmitted illness presentation to high-school students, the nurse recommends the human papillomavirus (HPV) vaccine series. Which condition is the nurse trying to prevent?

Cervical cancer

Having misplaced a stethoscope, a nurse borrows a colleague's stethoscope. The nurse next enters the patient's room and identifies self, washes hands with soap, and states the purpose of the visit. The nurse performs proper identification of the patient before auscultating the patient's lungs. Which critical health assessment step should the nurse have performed?

Cleaning stethoscope with alcohol

The nurse completed assessments on several patients. Which assessment finding will the nurse record as normal?

Constricting pupils when directly illuminated

A school nurse recognizes a belt buckle-shaped ecchymosis on a 7-year-old student. When privately asked about how the injury occurred, the student described falling on the playground. Which action will the nurse take next?

Contact social services and report suspected abuse.

A diabetic patient presents to the clinic for a dressing change. The wound is located on the right foot and has purulent yellow drainage. Which action will the nurse take to prevent the spread of infection? a.Position the patient comfortably on the stretcher. b.Explain the procedure for dressing change to the patient. c.Review the medication list that the patient brought from home. d.Don gloves and other appropriate personal protective equipment.

D

The nurse is caring for a patient on contact precautions. Which action will be most appropriate to prevent the spread of disease? a.Place the patient in a room with negative airflow. b.Wear a gown, gloves, face mask, and goggles for interactions with the patient. c.Transport the patient safely and quickly when going to the radiology department. d.Use a dedicated blood pressure cuff that stays in the room and is used for that patient only.

D

The nurse is caring for a patient who is at risk for infection. Which action by the nurse indicates correct understanding about standard precautions? a.Teaches the patient about good nutrition b.Dons gloves when wearing artificial nails c.Disposes an uncapped needle in the designated container d.Wears eyewear when emptying the urinary drainage bag

D

The nurse is caring for a patient who needs a protective environment. The nurse has provided the care needed and is now leaving the room. In which order will the nurse remove the personal protective equipment, beginning with the first step? 1. Remove eyewear/face shield and goggles. 2. Perform hand hygiene, leave room, and close door. 3. Remove gloves. 4. Untie gown, allow gown to fall from shoulders, and do not touch outside of gown; dispose of properly. 5. Remove mask by strings; do not touch outside of mask. 6. Dispose of all contaminated supplies and equipment in designated receptacles. a.3, 1, 4, 5, 6, 2 b.1, 4, 5, 3, 6, 2 c.1, 4, 5, 3, 2, 6 d.3, 1, 4, 5, 2, 6

D

The nurse is caring for a school-aged child who has injured the right leg after a bicycle accident. Which signs and symptoms will the nurse assess for to determine if the child is experiencing a localized inflammatory response? a.Malaise, anorexia, enlarged lymph nodes, and increased white blood cells b.Chest pain, shortness of breath, and nausea and vomiting c.Dizziness and disorientation to time, date, and place d.Edema, redness, tenderness, and loss of function

D

The nurse is performing hand hygiene before assisting a health care provider with insertion of a chest tube. While washing hands, the nurse touches the sink. Which action will the nurse take next? a.Inform the health care provider and recruit another nurse to assist. b.Rinse and dry hands, and begin assisting the health care provider. c.Extend the handwashing procedure to 5 minutes. d.Repeat handwashing using antiseptic soap.

D

The nurse is providing an educational session for a group of preschool workers. The nurse reminds the group about the most important thing to do to prevent the spread of infection. Which information did the nurse share with the preschool workers? a.Encourage preschool children to eat a nutritious diet. b.Suggest that parents provide a multivitamin to the children. c.Clean the toys every afternoon before putting them away. d.Wash their hands between each interaction with children.

D

The nurse manager is evaluating current infection control data for the intensive care unit. The nurse compares past patient data with current data to look for trends. The nurse manager examines the infection chain for possible solutions. In which order will the nurse arrange the items for the infection chain beginning with the first step? 1. A mode of transmission 2. An infectious agent or pathogen 3. A susceptible host 4. A reservoir or source for pathogen growth 5. A portal of entry to a host 6. A portal of exit from the reservoir a.3, 2, 4, 1, 5, 6 b.1, 3, 5, 4, 6, 2 c.4, 2, 1, 6, 3, 5 d.2, 4, 6, 1, 5, 3

D

Which interventions utilized by the nurse will indicate the ability to recognize a localized inflammatory response? a.Vigorous range-of-motion exercises b.Turn, cough, and deep breathe c.Orient to date, time, and place d.Rest, ice, and elevation

D

Having misplaced a stethoscope, a nurse borrows a colleague's stethoscope. The nurse next enters the patient's room and identifies self, washes hands with soap, and states the purpose of the visit. The nurse performs proper identification of the patient before auscultating the patient's lungs. Which critical health assessment step should the nurse have performed? A. Running warm water over stethoscope B. Draping stethoscope around the neck C. Rubbing stethoscope with betadine D. Cleaning stethoscope with alcohol

D. Cleaning stethoscope with alcohol Bacteria and viruses can be transferred from patient to patient when a stethoscope that is not clean is used. The stethoscope should be cleaned before use on each patient with isopropyl alcohol. Running water over the stethoscope does not kill bacteria. Betadine is an inappropriate cleaning solution and may damage the equipment. Draping the stethoscope around the neck is not advised.

A school nurse recognizes a belt buckle-shaped ecchymosis on a 7-year-old student. When privately asked about how the injury occurred, the student described falling on the playground. Which action will the nurse take next? A. Talk to the principal about how to proceed. B. Disregard the finding based upon child's response. C. Interview the patient in the presence of the teacher. D. Contact social services and report suspected abuse.

D. Contact social services and report suspected abuse. Most states mandate a report to a social service center if nurses suspect abuse or neglect. When abuse is suspected, the nurse interviews the patient in private, not with a teacher. Observe the behavior of the individual for any signs of frustration, explanations that do not fit his or her physical presentation, or signs of injury. The nurse knows how to proceed and does not need to talk to the principal about what to do. Disregarding the finding is not advised because victims often will not complain or report that they are in an abusive situation.

During a routine pediatric history and physical, the parents report that their child was a very small, premature infant that had to stay in the neonatal intensive care unit longer than usual. They state that the infant was yellow when born and developed an infection that required "every antibiotic under the sun" to reach a cure. Which exam is a priority for the nurse to conduct on the child? A. Cardiac B. Respiratory C. Ophthalmic D. Hearing acuity

D. Hearing acuity Hearing is the priority. Risk factors for hearing problems include low birth weight, nonbacterial intrauterine infection, and excessively high bilirubin levels. Hearing loss due to ototoxicity (injury to auditory nerves) can result from high maintenance doses of antibiotics. Cardiac, respiratory, and eye examinations are important assessments but are not relevant to this child's condition.

The nurse is urgently called to the gymnasium regarding an injured student. The student is crying in severe pain with a malformed fractured lower leg. Which proper sequence will the nurse follow to perform the initial assessment? A. Light palpation, deep palpation, and inspection B. Inspection, light palpation, and deep palpation C. Auscultation and light palpation D. Inspection and light palpation

D. Inspection and light palpation Inspection is the use of vision and hearing to distinguish normal from abnormal findings. Light palpation determines areas of tenderness and skin temperature, moisture, and texture. Deep palpation is used to examine the condition of organs, such as those in the abdomen. Caution is the rule with deep palpation. Deep palpation is performed after light palpation; however, deep palpation is not performed on a fractured leg. Auscultation is used to evaluate sound and is not used to assess a fractured leg.

A patient in the emergency department is reporting left lower abdominal pain. Which proper order will the nurse follow to perform the comprehensive abdominal examination? A. Percussion, palpation, auscultation B. Percussion, auscultation, palpation C. Inspection, palpation, auscultation D. Inspection, auscultation, palpation

D. Inspection, auscultation, palpation The order of an abdominal examination differs slightly from that of other assessments. Begin with inspection and follow with auscultation. By using auscultation before palpation, the chance of altering the frequency and character of bowel sounds is lessened.

Upon assessment, the patient is breathing normally and has normal vesicular lung sounds. Which expected inspiratory-to-expiratory breath sounds will the nurse hear? A. The expiration phase is longer than the inspiration phase. B. The inspiratory phase lasts exactly as long as the expiratory phase. C. The expiration phase is 2 times longer than the inspiration phase. D. The inspiratory phase is 3 times longer than the expiratory phase.

D. The inspiratory phase is 3 times longer than the expiratory phase. Vesicular breath sounds are normal breath sounds; the inspiratory phase is 3 times longer than the expiratory phase. Bronchovesicular breath sounds have an inspiratory phase equal to the expiratory phase. Bronchial breath sounds have an expiration phase longer than the inspiration phase at a 3:2 ratio.

A head and neck physical examination is completed on a 50-year-old female patient. All physical findings are normal except for fine brittle hair. Which laboratory test will the nurse expect to be ordered, based upon the physical findings? A. Oxygen saturation B. Liver function test C. Carbon monoxide D. Thyroid-stimulating hormone test

D. Thyroid-stimulating hormone test Thyroid disease can make hair thin and brittle. Liver function testing is indicated for a patient who has jaundice. Oxygen saturation will be used for cyanosis. Cherry-colored lips indicate carbon monoxide poisoning.

A nurse is providing care to a group of patients. Which patient will the nurse see first? A) A bedridden patient who has a reddened area on the buttocks who needs to be turned. B) A patient after knee surgery who needs range of motion exercises. C) A patient on bed rest who has renal calculi and needs to go to the bathroom. D) A patient with a hip replacement on prolonged bed rest reporting chest pain and dyspnea.

D: ABCs (airway, breathing, circulation) dyspnea is difficulty breathing

A patient in the emergency department is reporting left lower abdominal pain. Which proper order will the nurse follow to perform the comprehensive abdominal examination? A) Percussion, palpation, auscultation, inspection B) Percussion, auscultation, inspection, palpation C) Inspection, palpation, auscultation, percussion D) Inspection, auscultation, percussion, palpation

D: others can alter bowel sounds, rupture a AAA (abdominal aortic aneurysm)

The patient is a 45-year-old African-American male who has come in for a routine annual physical. Which type of preventive screening does the nurse discuss with the patient?

Digital rectal examination of the prostate

A teen patient is tearful and reports locating lumps in her breasts. Other history obtained is that she is currently menstruating. Physical examination reveals soft and movable cysts in both breasts that are painful to palpation. The nurse also notes that the patient's nipples are erect, but the areola is wrinkled. Which action will the nurse take after talking with the health care provider?

Discuss the possibility of fibrocystic disease as the probable cause.

While assessing the skin of an 82-year-old patient, a nurse discovers nonpainful, ruby red papules on the patient's trunk. What is the nurse's next action?

Document cherry angiomas as a normal older adult skin finding.

An older-adult patient is taking aminoglycoside for a severe infection. Which assessment is the priority?

Ears

The patient has had a stroke that has affected the ability to speak. The patient becomes extremely frustrated when trying to speak. The patient responds correctly to questions and instructions but cannot form words coherently. Which type of aphasia is the patient experiencing?

Expressive

On admission, a patient weighs 250 pounds. The weight is recorded as 256 pounds on the second inpatient day. Which condition will the nurse assess for in this patient?

Fluid retention

The nurse is assessing skin turgor. Which technique will the nurse use?

Grasp a fold of skin on the sternal area.

A male student comes to the college health clinic. He hesitantly describes that he found something wrong with his testis when taking a shower. Which assessment finding will alert the nurse to possible testicular cancer?

Hard, pea-sized testicular lump

During a routine pediatric history and physical, the parents report that their child was a very small, premature infant that had to stay in the neonatal intensive care unit longer than usual. They state that the infant was yellow when born and developed an infection that required "every antibiotic under the sun" to reach a cure. Which exam is a priority for the nurse to conduct on the child?

Hearing acuity

An older-adult patient is being seen for chronic entropion. Which condition will the nurse assess for in this patient?

Infection

The nurse is urgently called to the gymnasium regarding an injured student. The student is crying in severe pain with a malformed fractured lower leg. Which proper sequence will the nurse follow to perform the initial assessment?

Inspection and light palpation

A patient in the emergency department is reporting left lower abdominal pain. Which proper order will the nurse follow to perform the comprehensive abdominal examination?

Inspection, auscultation, palpation

The nurse is examining a female with vaginal discharge. Which position will the nurse place the patient for proper examination?

Lithotomy

A nurse is assessing several patients. Which assessment findings will cause the nurse to follow up? (Select all that apply.)

Orthopnea - Pleural friction rub present - Crackles in lower lung lobes

During a school physical examination, the nurse reviews the patient's current medical history. The nurse discovers the patient has allergies. Which assessment finding is consistent with allergies?

Pale nasal mucosa

The patient presents to the clinic with dysuria and hematuria. How does the nurse proceed to assess for kidney inflammation?

Percusses posteriorly the costovertebral angle at the scapular line.

A nurse is conducting Weber's test. Which action will the nurse take?

Place a vibrating tuning fork in the middle of patient's forehead.

During a genitourinary examination of a 30-year-old male patient, the nurse identifies a small amount of a white, thick substance on the patient's uncircumcised glans penis. What is the nurse's next step?

Record this as a normal finding.

The nurse is preparing for a rectal examination of a nonambulatory male patient. In which position will the nurse place the patient?

Sims'

A nurse is preparing to perform a complete physical examination on a weak, older-adult patient with bilateral basilar pneumonia. Which position will the nurse use?

Supine

Upon assessment, the patient is breathing normally and has normal vesicular lung sounds. Which expected inspiratory-to-expiratory breath sounds will the nurse hear?

The inspiratory phase is 3 times longer than the expiratory phase.

A head and neck physical examination is completed on a 50-year-old female patient. All physical findings are normal except for fine brittle hair. Which laboratory test will the nurse expect to be ordered, based upon the physical findings?

Thyroid-stimulating hormone test

The nurse is assessing the tympanic membranes of an infant. Which action by the nurse demonstrates proper technique?

Uses an inverted otoscope grip while pulling the auricle downward and back.

A nurse is auscultating different areas on an adult patient. Which technique should the nurse use during an assessment?

Uses the diaphragm to listen for bowel sounds

A febrile preschool-aged child presents to the after-hours clinic. Varicella (chickenpox) is diagnosed on the basis of the illness history and the presence of small, circumscribed skin lesions filled with serous fluid. Which type of skin lesion will the nurse report?

Vesicles

A nurse is assessing a patient's cranial nerve IX. Which items does the nurse gather before conducting the assessment? (Select all that apply.)

Vial of sugar - Tongue blade - Lemon applicator

A nurse identifies lice during a child's scalp assessment. The nurse teaches the parents about hair care. Which information from the parents indicates the nurse needs to follow up?

We will use lindane-based shampoos.

A parent calls the school nurse with questions regarding the recent school vision screening. Snellen chart examination revealed 20/60 for both eyes. Which response by the nurse is the best regarding the eye examination results?

Your child needs to see an ophthalmologist.

A nurse is a preceptor for a nurse who just graduated from nursing school. When caring for a patient, the new graduate nurse begins to explain to the patient the purpose of completing a physical assessment. Which statement made by the new graduate nurse requires the preceptor to intervene?

"Nursing assessment data are used only to provide information about the effectiveness of your medical care.

A nurse is assessing a group of patients. Match the assessment finding the nurse observed to its condition.

1. Koilonychia - Spoon nails 2. Venous problems - Lower extremity swollen and warm with normal pulse 3. Lordosis - Swayback 4. Melena - Black, tarry stools 5. Arterial problems - Lower extremity pale and cool with decreased pulse 6. Jugular vein distention - Neck vein visible when sitting 7. Tinnitus- Ringing in ears

The nurse is assessing an adult patient's patellar reflex. Which finding will the nurse record as normal?

2+

The nurse considers several new female patients to receive additional teaching on the need for more frequent Pap test and gynecological examinations. Which assessment findings reveal the patient at highest risk for cervical cancer and having the greatest need for patient education?

22 years old, smokes 1 pack of cigarettes per day, has multiple sexual partners

A nurse is preparing to perform a lung assessment on a patient and discovers through the nursing history the patient smokes. The nurse figures the pack-years for this patient who has smoked two and a half (2 1/2) packs a day for 20 years. Which value will the nurse record in the patient's medical record? Record answer as a whole number. _________ pack-years

50 Pack-years = Number of years smoking×Number of packs per day: 20×2.5 = 50.

A nurse is preparing to perform a lung assessment on a patient and discovers through the nursing history the patient smokes. The nurse figures the pack-years for this patient who has smoked two and a half (2 1/2) packs a day for 20 years. Which value will the nurse record in the patient's medical record? Record answer as a whole number. _________ pack-years

50 (Pack-years = Number of years smoking × Number of packs per day: 20 × 2.5 = 50.)

The paramedics transport an adult involved in a motor vehicle accident to the emergency department. On physical examination, the patient's level of consciousness is reported as opening eyes to pain and responding with inappropriate words and flexion withdrawal to painful stimuli. Which value will the nurse report for the patient's Glasgow Coma Scale score?

9

A patient presents with pneumonia. Which priority intervention should be included in the plan of care for this patient? a.Observe the patient for decreased activity tolerance. b.Assume the patient is in pain and treat accordingly. c.Provide the patient ice chips as requested. d.Maintain the room temperature at 65° F.

A

The nurse and a new nurse in orientation are caring for a patient with pneumonia. Which statement by the new nurse will indicate a correct understanding of this condition? a."An infectious disease like pneumonia may not pose a risk to others." b."We need to isolate the patient in a private negative-pressure room." c."Clinical signs and symptoms are not present in pneumonia." d."The patient will not be able to return home."

A

The nurse is caring for a group of patients. Which patient will the nurse see first? a.A patient with Clostridium difficile in droplet precautions b.A patient with tuberculosis in airborne precautions c.A patient with MRSA infection in contact precautions d.A patient with a lung transplant in protective environment precautions

A

The nurse is caring for a patient in the hospital. The nurse observes the nursing assistive personnel (NAP) turning off the handle faucet with bare hands. Which professional practice principle supports the need for follow-up with the NAP? a.The nurse is responsible for providing a safe environment for the patient. b.Different scopes of practice allow modification of procedures. c.Allowing the water to run is a waste of resources and money. d.This is a key step in the procedure for washing hands.

A

The nurse is caring for a patient who becomes nauseated and vomits without warning. The nurse has contaminated hands. Which action is best for the nurse to take next? a.Wash hands with an antimicrobial soap and water. b.Clean hands with wipes from the bedside table. c.Use an alcohol-based waterless hand gel. d.Wipe hands with a dry paper towel.

A

The nurse is caring for a patient who has cultured positive for Clostridium difficile. Which action will the nurse take next? a.Instruct assistive personnel to use soap and water rather than sanitizer. b.Wear an N95 respirator when entering the patient room. c.Place the patient on droplet precautions. d.Teach the patient cough etiquette.

A

The nurse is dressed and is preparing to care for a patient in the perioperative area. The nurse has scrubbed hands and has donned a sterile gown and gloves. Which action will indicate a break in sterile technique? a.Touching clean protective eyewear b.Standing with hands above waist area c.Accepting sterile supplies from the surgeon d.Staying with the sterile table once it is open

A

The nurse is observing a family member changing a dressing for a patient in the home health environment. Which observation indicates the family member has a correct understanding of how to manage contaminated dressings? a.The family member places the used dressings in a plastic bag. b.The family member saves part of the dressing because it is clean. c.The family member removes gloves and gathers items for disposal. d.The family member wraps the used dressing in toilet tissue before placing in trash.

A

The nurse on the surgical team and the surgeon have completed a surgery. After donning gloves, gathering instruments, and placing in the transport carrier, what is the next step in handling the instruments used during the procedure? a.Sending to central sterile for cleaning and sterilization b.Sending to central sterile for cleaning and disinfection c.Sending to central sterile for cleaning and boiling d.Sending to central sterile for cleaning

A

The surgical mask the perioperative nurse is wearing becomes moist. Which action will the perioperative nurse take next? a.Apply a new mask. b.Reapply the mask after it air-dries. c.Change the mask when relieved by next shift. d.Do not change the mask if the nurse is comfortable.

A

A nurse is assessing a group of patients. Match the assessment finding the nurse observed to its condition: A. Lower extremity swollen and warm with normal pulse B. Neck vein visible when sitting C. Spoon nails D. Lower extremity pale and cool with decreased pulse E. Ringing in ears F. Swayback G. Black, tarry stools 1. Koilonychia 2. Venous problems 3. Lordosis 4. Melena 5. Arterial problems 6. Jugular vein distention 7. Tinnitus

A - 2 B - 6 C - 1 D - 5 E - 7 F - 3 G - 4

A nurse is caring for a group of patients. Which patient will the nurse see first? A. An adult with an S4 heart sound B. A young adult with an S3 heart sound C. An adult with vesicular lung sounds in the lung periphery D. A young adult with bronchovesicular breath sounds between the scapula posteriorly

A. An adult with an S4 heart sound A fourth heart sound (S4) occurs when the atria contract to enhance ventricular filling. An S4 is often heard in healthy older adults, children, and athletes, but it is not normal in adults. Because S4 also indicates an abnormal condition, report it to a health care provider. An S3 is considered abnormal in adults over 31 years of age but can often be heard normally in children and young adults. Vesicular lungs sounds in the periphery and bronchovesicular lung sounds in between the scapula are normal findings.

During a routine physical examination of a 70-year-old patient, a blowing sound is auscultated over the carotid artery. Which assessment finding will the nurse report to the health care provider? A. Bruit B. Thrill C. Phlebitis D. Right-sided heart failure

A. Bruit A bruit is the sound of turbulence of blood passing through a narrowed blood vessel and is auscultated as a blowing sound. A bruit can reflect cardiovascular disease in the carotid artery of middle-aged to older adults. Intensity or loudness is related to the rate of blood flow through the heart or the amount of blood regurgitated. A thrill is a continuous palpable sensation that resembles the purring of a cat. Jugular venous distention, not bruit, is a possible sign of right-sided heart failure. Some patients with heart disease have distended jugular veins when sitting. Phlebitis is an inflammation of a vein that occurs commonly after trauma to the vessel wall, infection, immobilization, and prolonged insertion of IV catheters. It affects predominantly peripheral veins.

The patient is a 45-year-old African-American male who has come in for a routine annual physical. Which type of preventive screening does the nurse discuss with the patient? A. Digital rectal examination of the prostate B. Complete eye examination every year C. CA 125 blood test once a year D. Colonoscopy every 3 years

A. Digital rectal examination of the prostate Recommended preventive screenings include a digital rectal examination of the prostate and prostate-specific antigen test starting at age 50. CA 125 blood tests are indicated for women at high risk for ovarian cancer. Patients over the age of 65 need to have complete eye examinations yearly. Colonoscopy every 10 years is recommended in patients 50 years of age and older.

A male student comes to the college health clinic. He hesitantly describes that he found something wrong with his testis when taking a shower. Which assessment finding will alert the nurse to possible testicular cancer? A. Hard, pea-sized testicular lump B. Rubbery texture of testes C. Painful enlarged testis D. Prolonged diuretic use

A. Hard, pea-sized testicular lump The most common symptoms of testicular cancer are a painless enlargement of one testis and the appearance of a palpable, small, hard lump, about the size of a pea, on the front or side of the testicle. Normally, the testes feel smooth, rubbery, and free of nodules. Use of diuretics, sedatives, or antihypertensives can lead to erection or ejaculation problems.

A nurse is conducting Weber's test. Which action will the nurse take? A. Place a vibrating tuning fork in the middle of patient's forehead. B. Place a vibrating tuning fork on the patient's mastoid process. C. Compare the number of seconds heard by bone versus air conduction. D. Compare the patient's degree of joint movement to the normal level.

A. Place a vibrating tuning fork in the middle of patient's forehead. During Weber's test (lateralization of sound), the nurse places the vibrating tuning fork in the middle of the patient's forehead. During a Rinne test (comparison of air and bone conduction), the nurse places a vibrating tuning fork on the patient's mastoid process and compares the length of time air and bone conduction is heard. Comparing the patient's degree of joint movement to the normal level is a test for range of motion.

During a genitourinary examination of a 30-year-old male patient, the nurse identifies a small amount of a white, thick substance on the patient's uncircumcised glans penis. What is the nurse's next step? A. Record this as a normal finding. B. Avoid embarrassing questions about sexual activity. C. Notify the provider about a suspected sexually transmitted infection. D. Tell the patient to avoid doing self-examinations until symptoms clear.

A. Record this as a normal finding. A small amount of thick, white smegma sometimes collects under the foreskin in the uncircumcised male and is considered normal. Penile pain or swelling, genital lesions, and urethral discharge are signs and symptoms that may indicate sexually transmitted infections (STI). All men 15 years and older need to perform a male-genital self-examination monthly. The nurse needs to assess a patient's sexual history and use of safe sex habits. Sexual history reveals risks for STI and HIV.

The nurse is preparing for a rectal examination of a nonambulatory male patient. In which position will the nurse place the patient? A. Sims' B. Knee-chest C. Dorsal recumbent D. Forward bending with flexed hips

A. Sims' Nonambulatory patients are best examined in a side-lying Sims' position. Forward bending would require the patient to be able to stand upright. Knees to chest would be difficult to maintain in a nonambulatory male and is embarrassing and uncomfortable. Dorsal recumbent does not provide adequate access for a rectal examination and is used for abdominal assessment because it promotes relaxation of abdominal muscles.

A febrile preschool-aged child presents to the after-hours clinic. Varicella (chickenpox) is diagnosed on the basis of the illness history and the presence of small, circumscribed skin lesions filled with serous fluid. Which type of skin lesion will the nurse report? A. Vesicles B. Wheals C. Papules D. Pustules

A. Vesicles Vesicles are circumscribed, elevated skin lesions filled with serous fluid that measure less than 1 cm. Wheals are irregularly shaped, elevated areas of superficial localized edema that vary in size. They are common with mosquito bites and hives. Papules are palpable, circumscribed, solid elevations in the skin that are smaller than 1 cm. Pustules are elevations of skin similar to vesicles, but they are filled with pus and vary in size like acne.

A nurse identifies lice during a child's scalp assessment. The nurse teaches the parents about hair care. Which information from the parents indicates the nurse needs to follow up? A. We will use lindane-based shampoos. B. We will use the sink to wash hair. C. We will use a fine-toothed comb. D. We will use a vinegar hair rinse.

A. We will use lindane-based shampoos. Products containing lindane, a toxic ingredient, often cause adverse reactions; the nurse will need to follow up to correct the misconception. All the rest are correct. Instruct parents who have children with head lice to shampoo thoroughly with pediculicide (shampoo available at drugstores) in cold water at a basin or sink, comb thoroughly with a fine-toothed comb, and discard the comb. A dilute solution of vinegar and water helps loosen nits.

A parent calls the school nurse with questions regarding the recent school vision screening. Snellen chart examination revealed 20/60 for both eyes. Which response by the nurse is the best regarding the eye examination results? A. Your child needs to see an ophthalmologist. B. Your child is suffering from strabismus. C. Your child may have presbyopia. D. Your child has cataracts.

A. Your child needs to see an ophthalmologist. The child needs an eye examination with an ophthalmologist or optometrist. Normal vision is 20/20. The larger the denominator, the poorer the patient's visual acuity. For example, a value of 20/60 means that the patient, when standing 20 feet away, can read a line that a person with normal vision can read from 60 feet away. Strabismus is a (congenital) condition in which both eyes do not focus on an object simultaneously: The eyes appear crossed. Acuity may not be affected; Snellen test does not test for strabismus. Presbyopia is impaired near vision that occurs in middle-aged and older adults and is caused by loss of elasticity of the lens. Cataracts, a clouding of the lens, develop slowly and progressively after age 35 or suddenly after trauma.

The patient has been in bed for several days and needs to be ambulated. Which action will the nurse take first? A) Dangle the patient at the bedside. B) Maintain a narrow base of support. C) Encourage isometric exercises. D) Suggest a high-calcium diet.

A: help with blood pressure and circulation to stabilize, shoulder width base support, active ROM, low calcium diet (already at risk for hypercalcemia due to being in bed for several days (bone resorption))

The nurse is caring for an older-adult patient who has been diagnosed with a stroke. Which intervention will the nurse add to the care plan? A) Encourage the patient to perform as many self-care activities as possible. B) Provide a complete bed bath to promote patient comfort. C) Coordinate with occupational therapy for gait training. D) Place the patient on bed rest to prevent fatigue.

A: inc. activity (patient should be able to assist in some way with bathing, due to hemiparesis/hemiparalysis), PT does gait training (gross motor), OT fine motor adaptive devices, ADLs; doesn't need to be put on bedrest (doesn't prevent fatigue)

The nurse is caring for a patient who refuses to bathe in the morning. When asked why, the patient says "I always bathe in the evening." Which action by the nurse is best? A) Defer the bath until evening and pass on the information to the next shift. B) Tell the patient that daily morning baths are the "normal" routine. C) Explain the importance of maintaining morning hygiene practices. D) Cancel hygiene for the day and attempt again in the morning.

A: try to stick to patient's own routine and request (daily morning baths are not normal routine for everyone), therapeutic communication

During a routine physical examination of a 70-year-old patient, a blowing sound is auscultated over the carotid artery. Which assessment finding will the nurse report to the health care provider? A) Bruit B) Thrill C) Phlebitis D) Right sided heart failure

A: turbulence; thrill (you feel), phlebitis (erythema, edema) visual at a vein when there is irritation; heart failure: crackles in the lungs (more so with left sided HF) or JVD (right sided heart failure)

1. A nurse is preparing to perform a lung assessment on a patient and discovers through the nursing history the patient smokes. The nurse figures the pack-years for this patient who has smoked two and a half (2 1/2) packs a day for 20 years. Which value will the nurse record in the patient's medical record? Record answer as a whole number. _________ pack-years

ANS: 50 Pack-years = Number of years smoking × Number of packs per day: 20 × 2.5 = 50.

26. During a routine physical examination of a 70-year-old patient, a blowing sound is auscultated over the carotid artery. Which assessment finding will the nurse report to the health care provider? a. Bruit b. Thrill c. Phlebitis d. Right-sided heart failure

ANS: A A bruit is the sound of turbulence of blood passing through a narrowed blood vessel and is auscultated as a blowing sound. A bruit can reflect cardiovascular disease in the carotid artery of middle-aged to older adults. Intensity or loudness is related to the rate of blood flow through the heart or the amount of blood regurgitated. A thrill is a continuous palpable sensation that resembles the purring of a cat. Jugular venous distention, not bruit, is a possible sign of right-sided heart failure. Some patients with heart disease have distended jugular veins when sitting. Phlebitis is an inflammation of a vein that occurs commonly after trauma to the vessel wall, infection, immobilization, and prolonged insertion of IV catheters. It affects predominantly peripheral veins.

30. A nurse is caring for a group of patients. Which patient will the nurse see first? a. An adult with an S4 heart sound b. A young adult with an S3 heart sound c. An adult with vesicular lung sounds in the lung periphery d. A young adult with bronchovesicular breath sounds between the scapula posteriorly

ANS: A A fourth heart sound (S4) occurs when the atria contract to enhance ventricular filling. An S4 is often heard in healthy older adults, children, and athletes, but it is not normal in adults. Because S4 also indicates an abnormal condition, report it to a health care provider. An S3 is considered abnormal in adults over 31 years of age but can often be heard normally in children and young adults. Vesicular lungs sounds in the periphery and bronchovesicular lung sounds in between the scapula are normal findings.

A nurse is supervising the logrolling of a patient. To which patient is the nurse most likely providing care? a.A patient with neck surgery b.A patient with hypostatic pneumonia c.A patient with a total knee replacement d.A patient with a Stage IV pressure ulcer

ANS: A A nurse supervises and aids personnel when there is a health care provider's order to logroll a patient. Patients who have suffered from spinal cord injury or are recovering from neck, back, or spinal surgery often need to keep the spinal column in straight alignment to prevent further injury. Hypostatic pneumonia, total knee replacement, and Stage IV ulcers do not have to be logrolled.

A nurse is providing care to a group of patients. Which patient will the nurse see first? a.A patient with a hip replacement on prolonged bed rest reporting chest pain and dyspnea b.A bedridden patient who has a reddened area on the buttocks who needs to be turned c.A patient on bed rest who has renal calculi and needs to go to the bathroom d.A patient after knee surgery who needs range of motion exercises

ANS: A A patient on prolonged bed rest will be prone to deep vein thrombosis, which can lead to an embolus. An embolus can travel through the circulatory system to the lungs and impair circulation and oxygenation, resulting in tachycardia and shortness of breath. Venous emboli that travel to the lungs are sometimes life threatening. While the patient with a reddened area needs to be turned, a patient with renal calculi needing the restroom, and a patient needing range of motion, these are not as life threatening as the chest pain and dyspnea.

22. During a genitourinary examination of a 30-year-old male patient, the nurse identifies a small amount of a white, thick substance on the patient's uncircumcised glans penis. What is the nurse's next step? a. Record this as a normal finding. b. Avoid embarrassing questions about sexual activity. c. Notify the provider about a suspected sexually transmitted infection. d. Tell the patient to avoid doing self-examinations until symptoms clear.

ANS: A A small amount of thick, white smegma sometimes collects under the foreskin in the uncircumcised male and is considered normal. Penile pain or swelling, genital lesions, and urethral discharge are signs and symptoms that may indicate sexually transmitted infections (STI). All men 15 years and older need to perform a male-genital self-examination monthly. The nurse needs to assess a patient's sexual history and use of safe sex habits. Sexual history reveals risks for STI and HIV.

The patient has been diagnosed with a spinal cord injury and needs to be repositioned using the logrolling technique. Which technique will the nurse use for logrolling? a.Use at least three people. b.Have the patient reach for the opposite side rail when turning. c.Move the top part of the patient's torso and then the bottom part. d.Do not use pillows after turning.

ANS: A At least three to four people are needed to perform this skill safely. Have the patient cross the arms on the chest to prevent injury to the arms. Move the patient as one unit in a smooth, continuous motion on the count of three. Gently lean the patient as a unit back toward pillows for support.

The nurse is providing teaching to an immobilized patient with impaired skin integrity about diet. Which diet will the nurse recommend? a.High protein, high calorie b.High carbohydrate, low fat c.High vitamin A, high vitamin E d.Fluid restricted, bland

ANS: A Because the body needs protein to repair injured tissue and rebuild depleted protein stores, give the immobilized patient a high-protein, high-calorie diet. A high-carbohydrate, low-fat diet is not beneficial for an immobilized patient. Vitamins B and C are needed rather than A and E. Fluid restriction can be detrimental to the immobilized patient; this can lead to dehydration. A bland diet is not necessary for immobilized patients.

A patient has damage to the cerebellum. Which disorder is most important for the nurse to assess? a.Imbalance b.Hemiplegia c.Muscle sprain d.Lower extremity paralysis

ANS: A Damage to the cerebellum causes problems with balance, and motor impairment is directly related to the amount of destruction of the motor strip. A stroke can lead to hemiplegia. Direct trauma to the musculoskeletal system results in bruises, contusions, sprains, and fractures. A complete transection of the spinal cord can lead to lower extremity paralysis.

The patient is being admitted to the neurological unit with a diagnosis of stroke. When will the nurse begin discharge planning? a.At the time of admission b.The day before the patient is to be discharged c.When outpatient therapy will no longer be needed d.As soon as the patient's discharge destination is known

ANS: A Discharge planning begins when a patient enters the health care system. In anticipation of the patient's discharge from an institution, the nurse makes appropriate referrals or consults a case manager or a discharge planner to ensure that the patient's needs are met at home. Referrals to home care or outpatient therapy are often needed. Planning the day before discharge, when outpatient therapy is no longer needed, and as soon as the discharge destination is known is too late.

4. A nurse is conducting Weber's test. Which action will the nurse take? a. Place a vibrating tuning fork in the middle of patient's forehead. b. Place a vibrating tuning fork on the patient's mastoid process. c. Compare the number of seconds heard by bone versus air conduction. d. Compare the patient's degree of joint movement to the normal level.

ANS: A During Weber's test (lateralization of sound), the nurse places the vibrating tuning fork in the middle of the patient's forehead. During a Rinne test (comparison of air and bone conduction), the nurse places a vibrating tuning fork on the patient's mastoid process and compares the length of time air and bone conduction is heard. Comparing the patient's degree of joint movement to the normal level is a test for range of motion.

The nurse is assessing a patient's functional performance. What assessment parameters will be most important in this assessment? a. Continence assessment, gait assessment, feeding assessment, dressing assessment, transfer assessment b. Height, weight, body mass index (BMI), vital signs assessment c. Sleep assessment, energy assessment, memory assessment, concentration assessment d. Healthy individual, volunteers at church, works part time, takes care of family and house

ANS: A Functional impairment, disability, or handicap refers to varying degrees of an individual's inability to perform the tasks required to complete normal life activities without assistance. Height, weight, BMI, and vital signs are physical assessment. Sleep, energy, memory, and concentration are part of a depression screening. Healthy, volunteering, working, and caring for family and house are functional abilities, not performance.

A nurse delegates a position change to a nursing assistive personnel. The nurse instructs the NAP to place the patient in the lateral position. Which finding by the nurse indicates a correct outcome? a.Patient is lying on side. b.Patient is lying on back. c.Patient is lying semiprone. d.Patient is lying on abdomen.

ANS: A In the side-lying (or lateral) position the patient rests on the side with the major portion of body weight on the dependent hip and shoulder. Patients in the supine position rest on their backs. Sims' position is semiprone. The patient in the prone position lies face or chest down on the abdomen.

A nurse is conducting community education classes on skin cancer. One participant says to the nurse: "I read that most melanomas occur on the face and arms in fair-skinned women. Is this true?" The nurse's most helpful response would be which of the following? a. "That is not correct. Melanoma is more commonly found on the torso or the lower legs of women." b. "That is correct, because the face and arms are exposed more often to the sun." c. "That is not correct. Melanoma occurs on the top of the head in men but is rare in women." d. "That is incorrect. Melanoma is most commonly seen in dark-skinned individuals."

ANS: A Melanoma is more commonly found on the torso or the lower legs in women. Melanoma can occur anywhere and is not associated with direct exposure. For example, an individual can have melanoma under the skin and on the soles of the feet. Dark-skinned individuals are less likely to get melanoma.

23. The nurse is preparing for a rectal examination of a nonambulatory male patient. In which position will the nurse place the patient? a. Sims' b. Knee-chest c. Dorsal recumbent d. Forward bending with flexed hips

ANS: A Nonambulatory patients are best examined in a side-lying Sims' position. Forward bending would require the patient to be able to stand upright. Knees to chest would be difficult to maintain in a nonambulatory male and is embarrassing and uncomfortable. Dorsal recumbent does not provide adequate access for a rectal examination and is used for abdominal assessment because it promotes relaxation of abdominal muscles.

The nurse is caring for an older-adult patient who has been diagnosed with a stroke. Which intervention will the nurse add to the care plan? a.Encourage the patient to perform as many self-care activities as possible. b.Provide a complete bed bath to promote patient comfort. c.Coordinate with occupational therapy for gait training. d.Place the patient on bed rest to prevent fatigue.

ANS: A Nurses should encourage the older-adult patient to perform as many self-care activities as possible, thereby maintaining the highest level of mobility. Sometimes nurses inadvertently contribute to a patient's immobility by providing unnecessary help with activities such as bathing and transferring. Placing the patient on bed rest without sufficient ambulation leads to loss of mobility and functional decline, resulting in weakness, fatigue, and increased risk for falls. After a stroke or brain attack, a patient likely receives gait training from a physical therapist; speech rehabilitation from a speech therapist; and help from an occupational therapist for ADLs such as dressing, bathing and toileting, or household chores.

The nurse is caring for a patient who needs to be placed in the prone position. Which action will the nurse take? a.Place pillow under the patient's abdomen after turning. b.Turn head toward one side with large, soft pillow. c.Position legs flat against bed. d.Raise head of bed to 45 degrees.

ANS: A Placing a pillow under the patient's abdomen after turning decreases hyperextension of lumbar vertebrae and strain on lower back; breathing may also be enhanced. Head is turned toward one side with a small pillow to reduce flexion or hyperextension of cervical vertebrae. Legs should be supported with pillows to elevate toes and prevent footdrop. Forty-five degrees is the position for Fowler's position; prone is on the stomach.

8. A nurse identifies lice during a child's scalp assessment. The nurse teaches the parents about hair care. Which information from the parents indicates the nurse needs to follow up? a. We will use lindane-based shampoos. b. We will use the sink to wash hair. c. We will use a fine-toothed comb. d. We will use a vinegar hair rinse.

ANS: A Products containing lindane, a toxic ingredient, often cause adverse reactions; the nurse will need to follow up to correct the misconception. All the rest are correct. Instruct parents who have children with head lice to shampoo thoroughly with pediculicide (shampoo available at drugstores) in cold water at a basin or sink, comb thoroughly with a fine-toothed comb, and discard the comb. A dilute solution of vinegar and water helps loosen nits.

16. The patient is a 45-year-old African-American male who has come in for a routine annual physical. Which type of preventive screening does the nurse discuss with the patient? a. Digital rectal examination of the prostate b. Complete eye examination every year c. CA 125 blood test once a year d. Colonoscopy every 3 years

ANS: A Recommended preventive screenings include a digital rectal examination of the prostate and prostate-specific antigen test starting at age 50. CA 125 blood tests are indicated for women at high risk for ovarian cancer. Patients over the age of 65 need to have complete eye examinations yearly. Colonoscopy every 10 years is recommended in patients 50 years of age and older.

The nurse is assessing a patient's functional abilities and asks the patient, "How would you rate your ability to prepare a balanced meal?" "How would you rate your ability to balance a checkbook?" "How would you rate your ability to keep track of your appointments?" Which tool would be indicated for the best results of this patient's perception of their abilities? a. Functional Activities Questionnaire (FAQ)™ b. Mini Mental Status Exam (MMSE) c. 24hFAQ d. Performance-based functional measurement

ANS: A The FAQ is an example of a self-report tool which provides information about the patient's perception of functional ability. The MMSE assesses cognitive impairment. The 24hFAQ is used to assess functional ability in postoperative patients. Performance-based tools involve actual observation of a standardized task, completion of which is judged by objective criteria.

9. A parent calls the school nurse with questions regarding the recent school vision screening. Snellen chart examination revealed 20/60 for both eyes. Which response by the nurse is the best regarding the eye examination results? a. Your child needs to see an ophthalmologist. b. Your child is suffering from strabismus. c. Your child may have presbyopia. d. Your child has cataracts.

ANS: A The child needs an eye examination with an ophthalmologist or optometrist. Normal vision is 20/20. The larger the denominator, the poorer the patient's visual acuity. For example, a value of 20/60 means that the patient, when standing 20 feet away, can read a line that a person with normal vision can read from 60 feet away. Strabismus is a (congenital) condition in which both eyes do not focus on an object simultaneously: The eyes appear crossed. Acuity may not be affected; Snellen test does not test for strabismus. Presbyopia is impaired near vision that occurs in middle-aged and older adults and is caused by loss of elasticity of the lens. Cataracts, a clouding of the lens, develop slowly and progressively after age 35 or suddenly after trauma.

12. A male student comes to the college health clinic. He hesitantly describes that he found something wrong with his testis when taking a shower. Which assessment finding will alert the nurse to possible testicular cancer? a. Hard, pea-sized testicular lump b. Rubbery texture of testes c. Painful enlarged testis d. Prolonged diuretic use

ANS: A The most common symptoms of testicular cancer are a painless enlargement of one testis and the appearance of a palpable, small, hard lump, about the size of a pea, on the front or side of the testicle. Normally, the testes feel smooth, rubbery, and free of nodules. Use of diuretics, sedatives, or antihypertensives can lead to erection or ejaculation problems.

A nurse is developing an individualized plan of care for a patient. Which action is important for the nurse to take? a.Establish goals that are measurable and realistic. b.Set goals that are a little beyond the capabilities of the patient. c.Use the nurse's own judgment and not be swayed by family desires. d.Explain that without taking alignment risks, there can be no progress.

ANS: A The nurse must develop an individualized plan of care for each nursing diagnosis and must set goals that are individualized, realistic, and measurable. The nurse should set realistic expectations for care and should include the patient and family when possible. The goals focus on preventing problems or risks to body alignment and mobility.

A nurse is assessing body alignment. What is the nurse monitoring? a.The relationship of one body part to another while in different positions b.The coordinated efforts of the musculoskeletal and nervous systems c.The force that occurs in a direction to oppose movement d.The inability to move about freely

ANS: A The terms body alignment and posture are similar and refer to the positioning of the joints, tendons, ligaments, and muscles while standing, sitting, and lying. Body alignment means that the individual's center of gravity is stable. Body mechanics is a term used to describe the coordinated efforts of the musculoskeletal and nervous systems. Friction is a force that occurs in a direction to oppose movement. Immobility is the inability to move about freely.

6. A febrile preschool-aged child presents to the after-hours clinic. Varicella (chickenpox) is diagnosed on the basis of the illness history and the presence of small, circumscribed skin lesions filled with serous fluid. Which type of skin lesion will the nurse report? a. Vesicles b. Wheals c. Papules d. Pustules

ANS: A Vesicles are circumscribed, elevated skin lesions filled with serous fluid that measure less than 1 cm. Wheals are irregularly shaped, elevated areas of superficial localized edema that vary in size. They are common with mosquito bites and hives. Papules are palpable, circumscribed, solid elevations in the skin that are smaller than 1 cm. Pustules are elevations of skin similar to vesicles, but they are filled with pus and vary in size like acne.

A nurse is assessing the body alignment of a standing patient. Which finding will the nurse report as normal? a.When observed laterally, the spinal curves align in a reversed "S" pattern. b.When observed posteriorly, the hips and shoulders form an "S" pattern. c.The arms should be crossed over the chest or in the lap. d.The feet should be close together with toes pointed out.

ANS: A When the patient is observed laterally, the head is erect and the spinal curves are aligned in a reversed "S" pattern. When observed posteriorly, the shoulders and hips are straight and parallel. The arms hang comfortably at the sides. The feet are slightly apart to achieve a base of support, and the toes are pointed forward

34. A nurse is caring for a hospitalized patient with a urinary catheter. Which nursing action best prevents the patient from acquiring an infection? a. Maintaining a closed urinary drainage system b. Inserting the catheter using strict clean technique c. Disconnecting and replacing the catheter drainage bag once per shift d. Fully inflating the catheter's balloon according to the manufacturer's recommendation

ANS: A A key intervention to prevent infection is maintaining a closed urinary drainage system. A catheter should be inserted in the hospital setting using sterile technique. Inflating the balloon fully prevents dislodgment and trauma, not infection. Disconnecting the drainage bag from the catheter creates a break in the system and an open portal of entry and increases risk of infection.

4. A patient requests the nurse's help to the bedside commode and becomes frustrated when unable to void in front of the nurse. How should the nurse interpret the patient's inability to void? a. The patient can be anxious, making it difficult for abdominal and perineal muscles to relax enough to void. b. The patient does not recognize the physiological signals that indicate a need to void. c. The patient is lonely, and calling the nurse in under false pretenses is a way to get attention. d. The patient is not drinking enough fluids to produce adequate urine output.

ANS: A Attempting to void in the presence of another can cause anxiety and tension in the muscles that make voiding difficult. Anxiety can impact bladder emptying due to inadequate relaxation of the pelvic floor muscles and urinary sphincter. The nurse should give the patient privacy and adequate time if appropriate. No evidence suggests that an underlying physiological (does not recognize signals or not drinking enough fluids) or psychological (lonely) condition exists.

35. A nurse is providing care to a patient with an indwelling catheter. Which practice indicates the nurse is following guidelines for avoiding catheter-associated urinary tract infection (CAUTI)? a. Drapes the urinary drainage tubing with no dependent loops b. Washes the drainage tube toward the meatus with soap and water c. Places the urinary drainage bag gently on the floor below the patient d. Allows the spigot to touch the receptacle when emptying the drainage bag

ANS: A Avoid dependent loops in urinary drainage tubing. Prevent the urinary drainage bag from touching or dragging on the floor. When emptying the urinary drainage bag, use a separate measuring receptacle for each patient. Do not let the drainage spigot touch the receptacle. Using a clean washcloth, soap, and water, with your dominant hand wipe in a circular motion along the length of the catheter for about 10 cm (4 inches), starting at the meatus and moving away.

A nurse is teaching a community group of school-aged parents about safety. The most important item to prioritize and explain is how to check the proper fit of a. a bicycle helmet. b. swimming goggles. c. soccer shin guards. d. baseball sliding shorts.

ANS: A Bicycle-related injuries are a major cause of death and disability among children. Proper fit of the helmet helps to decrease head injuries resulting from bicycle accidents. Goggles, shin guards, and sliding shorts are important sports safety equipment and should fit properly, but they do not protect from this leading cause of death.

The nurse is caring for an elderly patient admitted with nausea, vomiting, and diarrhea. Upon completing the health history, which priority concern would require collaboration with social services to address the patient's health care needs? a. The electricity was turned off 2 days ago. b. The water comes from the county water supply. c. A son and family recently moved into the home. d. The home is not furnished with a microwave oven.

ANS: A Electricity is needed for refrigeration of food, and lack of electricity could have contributed to the nausea, vomiting, and diarrhea—potential food poisoning. This discussion about the patient's electrical needs can be referred to social services. The water supply, the increased number of individuals in the home, and not having a microwave may or may not be concerns but do not pertain to the current health care needs of this patient.

22. A patient has severe flank pain. The urinalysis reveals presence of calcium phosphate crystals. The nurse will anticipate an order for which diagnostic test? a. Intravenous pyelogram b. Mid-stream urinalysis c. Bladder scan d. Cystoscopy

ANS: A Flank pain and calcium phosphate crystals are associated with renal calculi. An intravenous pyelogram allows the provider to observe pathological problems such as obstruction of the ureter. A mid-stream urinalysis is performed for a routine urinalysis or if an infection is suspected; a urinalysis was already performed, a mid-stream would not be obtained again. A cystoscopy is used to detect bladder tumors and obstruction of the bladder outlet and urethra. A bladder scan measures the amount of urine in the bladder.

32. A nurse is caring for a patient with a continent urinary reservoir. Which action will the nurse take? a. Teach the patient how to self-cath the pouch. b. Teach the patient how to perform Kegel exercises. c. Teach the patient how to change the collection pouch. d. Teach the patient how to void using the Valsalva technique.

ANS: A In a continent urinary reservoir, the ileocecal valve creates a one-way valve in the pouch through which a catheter is inserted through the stoma to empty the urine from the pouch. Patients must be willing and able to catheterize the pouch 4 to 6 times a day for the rest of their lives. The second type of continent urinary diversion is called an orthotopic neobladder, which uses an ileal pouch to replace the bladder. Anatomically, the pouch is in the same position as the bladder was before removal, allowing a patient to void through the urethra using a Valsalva technique. In a ureterostomy or ileal conduit the patient has no sensation or control over the continuous flow of urine through the ileal conduit, requiring the effluent (drainage) to be collected in a pouch. Kegel exercises are ineffective for a patient with a continent urinary reservoir.

3. A patient is experiencing oliguria. Which action should the nurse perform first? a. Assess for bladder distention. b. Request an order for diuretics. c. Increase the patient's intravenous fluid rate. d. Encourage the patient to drink caffeinated beverages.

ANS: A Oliguria is diminished urinary output in relation to fluid intake. The nurse first should gather all assessment data to determine the potential cause of oliguria. It could be that the patient does not have adequate intake, or it could be that the bladder sphincter is not functioning and the patient is retaining water. Increasing fluids is effective if the patient does not have adequate intake or if dehydration occurs. Caffeine can work as a diuretic but is not helpful if an underlying pathology is present. An order for diuretics can be obtained if the patient was retaining water, but this should not be the first action.

14. A patient asks about treatment for stress urinary incontinence. Which is the nurse's bestresponse? a. Perform pelvic floor exercises. b. Avoid voiding frequently. c. Drink cranberry juice. d. Wear an adult diaper.

ANS: A Poor muscle tone leads to an inability to control urine flow. The nurse should recommend pelvic muscle strengthening exercises such as Kegel exercises; this solution best addresses the patient's problem. Evidence has shown that patients with urgency, stress, and mixed urinary incontinence can eventually achieve continence when treated with pelvic floor muscle training. Drinking cranberry juice is a preventative measure for urinary tract infection. The nurse should not encourage the patient to reduce voiding; residual urine in the bladder increases the risk of infection. Wearing an adult diaper could be considered if attempts to correct the root of the problem fail.

8. A patient has fallen several times in the past week when attempting to get to the bathroom. The patient gets up 3 or 4 times a night to urinate. Which recommendation by the nurse is most appropriate in correcting this urinary problem? a. Limit fluid and caffeine intake before bed. b. Leave the bathroom light on to illuminate a pathway. c. Practice Kegel exercises to strengthen bladder muscles. d. Clear the path to the bathroom of all obstacles before bedtime.

ANS: A Reducing fluids, especially caffeine and alcohol, before bedtime can reduce nocturia. To prevent nocturia, suggest that the patient avoid drinking fluids 2 hours before bedtime. Clearing a path to the bathroom, illuminating the path, or shortening the distance to the bathroom may reduce falls but will not correct the urination problem. Kegel exercises are useful if a patient is experiencing stress incontinence.

Equipment-related accidents are risks in the health care agency. The nurse assesses for this risk when using a. Sequential compression devices. b. A measuring device that measures urine. c. Computer-based documentation. d. A manual medication-dispensing device

ANS: A Sequential compression devices are used on a patient's extremities to assist in prevention of deep vein thrombosis and have the potential to malfunction and harm the patient. Measuring devices used by the nurse to measure urine, computer documentation, and manual dispensing devices can break or malfunction but are not used directly on a patient

The nurse is teaching a group of older adults at an assisted-living facility about age-related physiological changes. Which question would be the most important to ask this group? a. "Are you able to hear the tornado sirens in your area?" b. "Are you able to read your favorite book?" c. "Are you able to remember the name of the person you just met?" d. "Are you able to open a jar of pickles?"

ANS: A The ability to hear safety alerts and seek shelter is imperative to life safety. Although age-related changes may cause a decrease in sight that affects reading, and although difficulties in remembering short-term information and opening jars as arthritis sets in are important to patients and to those caring for them, being able to hear safety alerts is the priority.

The patient has been diagnosed with a respiratory illness and complains of shortness of breath. The nurse adjusts the temperature to facilitate the comfort of the patient. What is the usual comfort range for most patients? a. 65° F to 75° F b. 60° F to 75° F c. 15° C to 17° C d. 25° C to 28° C

ANS: A The comfort zone for most individuals is the range between 65° F and 75° F (18.3° C to 23.9° C). The other ranges do not reflect the average person's comfort zone.

A confused patient is restless and continues to try to remove his oxygen and urinary catheter. What is the priority nursing diagnosis and intervention to implement for this patient? a. Risk for injury: Prevent harm to patient, use restraints if alternatives fail. b. Deficient knowledge: Explain the purpose of oxygen therapy and the urinary catheter. c. Disturbed body image: Encourage patient to express concerns about body. d. Caregiver role strain: Identify resources to assist with care.

ANS: A The priority nursing diagnosis is risk for injury. This patient could cause harm to himself by interrupting the oxygen therapy or by damaging the urethra by pulling the urinary catheter out. Before restraining a patient, it is important to implement and exhaust alternatives to restraint. Alternatives can include distraction and providing companionship or supervision. Patients may be moved to a location closer to the nurses' station; trained sitters or family members may be involved. Nurses need to ensure that patients are provided adequate food, liquid, toileting, and relief from pain. If these and other alternatives fail, this individual may need restraints; in this case, an order would need to be obtained for the restraint. This patient may have deficient knowledge; educating the patient about treatments could be considered as an alternative to restraints; however, the nursing diagnosis of highest priority is risk for injury. This scenario does not indicate that the patient has a disturbed body image or that the patient's caregiver is strained.

The nurse is preparing a patient for surgery. The nurse explains that the reason for writing in indelible ink on the surgical site the word "correct" is to a. Distinguish the correct surgical site. b. Label the correct patient. c. Comply with the surgeon's preference. d. Adhere to the correct regulatory standard.

ANS: A The purpose of writing on the surgical site as part of the Universal Protocol from the Joint Commission is to distinguish the correct site on the correct patient and match with the correct surgeon for patient safety and prevention of wrong site surgery. All patients who are having an invasive procedure should receive labeling in many different ways, including the record and patient armbands. Writing in indelible ink may comply with the surgeon's preference, but safety is the driving factor. Although labeling of the site helps to meet regulatory standards, this is not the reason to do this activity—the reason is to keep the patient safe.

11. A nurse is planning care for a group of patients. Which task will the nurse assign to the nursing assistive personnel? a. Obtaining a midstream urine specimen b. Interpreting a bladder scan result c. Inserting a straight catheter d. Irrigating a catheter

ANS: A The skill of collecting midstream (clean-voided) urine specimens can be delegated to nursing assistive personnel. The nurse must first determine the timing and frequency of the bladder scan measurement and interprets the measurements obtained. Inserting a straight or an indwelling catheter cannot be delegated. Catheter irrigation or instillation cannot be delegated to nursing assistive personnel.

37. To reduce patient discomfort during a closed intermittent catheter irrigation, what should the nurse do? a. Use room temperature irrigation solution. b. Administer the solution as quickly as possible. c. Allow the solution to sit in the bladder for at least 1 hour. d. Raise the bag of the irrigation solution at least 12 inches above the bladder.

ANS: A To reduce discomfort use room temperature solution. Using cold solutions and instilling solutions too quickly can cause discomfort. During an irrigation, the solution does not sit in the bladder; it is allowed to drain. A container is not raised about the bladder 12 inches when performing a closed intermittent catheter irrigation.

33. The nurse is preparing to apply an external catheter. Which action will the nurse take? a. Allow 1 to 2 inches of space between the tip of the penis and the end of the catheter. b. Spiral wrap the penile shaft using adhesive tape to secure the catheter. c. Twist the catheter before applying drainage tubing to the end of the catheter. d. Shave the pubic area before applying the catheter.

ANS: A When applying an external catheter, allow 2.5 to 5 cm (1 to 2 inches) of space between the tip of the penis and the end of the catheter. Spiral wrap the penile shaft with supplied elastic adhesive. The strip should not overlap. The elastic strip should be snug but not tight. NOTE: Never use adhesive tape. Connect drainage tubing to the end of the condom catheter. Be sure the condom is not twisted. Connect the catheter to a large-volume drainage bag or leg. Clip hair at the base of the penile shaft, as necessary. Do not shave the pubic area.

The nurse is instructing the student nurse regarding discharge teaching and medications. Which response by the student would indicate that learning has occurred? a. "I need to be precise when teaching a patient about Zyprexa (olanzapine) and Zyrtec (cetirizine)." b. "The medications can be picked up at the pharmacy on the way out of the hospital." c. "I need to be sure to give the patient leftover medications from the medication drawer." d. "I need to remember to teach the patient to take all medications at the same time of the day."

ANS: A Zyprexa and Zyrtec are sound-alike, look-alike medications. Zyprexa is an antipsychotic and Zyrtec an antihistamine; these agents treat two different conditions. Bringing the differences and similarities in spelling and sound to the attention of the patient is important for patient safety. Medications are not distributed by the hospital, and medications do not need to be administered at the same time each day.

The nurse is assessing a patient's ability to perform basic activities of daily living (BADLs). Which of the following activities are considered in the BADLs assessment? (Select all that apply.) a. Brushing teeth or dentures b. Dressing oneself in the mornings c. Washing, drying, and folding laundry d. Counting own pulse and taking heart pill e. Taking the bus to the park f. Calling family members

ANS: A, B BADLs include actions related to self care and mobility and also includes eating, personal hygiene, and grooming activities. Instrumental activities of daily living (IADLs) include shopping, meal preparation, housekeeping, doing laundry, managing finances, using the telephone, taking medications, and using transportation.

A nurse is caring for an adult patient who has had a minor motor vehicle accident. The health history reveals that the patient is currently in the process of obtaining a divorce. Which of the following actions should the nurse take? (Select all that apply.) a. Agree upon and make time for the patient to talk. b. Use active listening skills and therapeutic touch as appropriate. c. Teach stress reduction strategies. d. Inform patient that stressed individuals are more likely to have accidents. e. Agree to witness telephone conversations with separated husband. f. Refer the patient to the nurse's church marriage counselor.

ANS: A, B, C, D Agreeing and making time for conversation, using active listening skills and therapeutic touch, teaching stress reduction strategies, and informing the patient of the risk to health associated with stress are interventions that are within the nurse's scope of practice. Agreeing to witness a telephone conversation could draw the nurse into divorce proceedings when the focus should be on the patient and his health. Referring the patient to the nurse's church counselor without a specific request from the patient may not take into consideration cultural care and could be considered unprofessional. If the patient requested a marriage counselor, a better solution would be to provide a referral to social services that may include a list of possible counselors from which the patient could choose.

The nurse is caring for a group of medical-surgical patients. The unit has been notified of a fire on an adjacent wing of the hospital. The nurse quickly formulates a plan to keep the patients safe. Which of the following should the nurse implement? (Select all that apply.) a. Close all doors. b. Note evacuation routes. c. Note oxygen shut-offs. d. Await direction from the fire department. e. Evacuate everyone from the building. f. Review "Stop, drop, and roll" with the nursing staff

ANS: A, B, C, D Closing all doors helps to contain smoke and fire. Noting the evacuation routes and oxygen shut-offs is important in case the direction to evacuate comes from established channels. Evacuation from the building is determined by the established chain of command or the fire department. Evacuation is done only when necessary. Review of "stop, drop, and roll," although important, is not a priority at this time.

The nurse suspects the possibility of a bioterrorist attack. Which of the following factors is most likely related to this possibility? (Select all that apply.) a. A rapid increase in patients presenting with fever or respiratory or gastrointestinal symptoms b. Lower rates of symptoms among patients who spend time primarily indoors c. Large number of rapidly fatal cases of patients with presenting symptoms d. Shortage of personal protective equipment available from central supply e. An increase in the number of staff calling in sick for their assigned shift f. Patients with symptoms all coming from one location in the area

ANS: A, B, C, F A rapid increase in patients presenting with a specific symptom, lower rates of symptoms among individuals indoors, and large numbers of fatalities with these symptoms all coming from one location are triggers that lead the nurse to suspect a bioterrorist attack. A shortage of personal protective equipment and an increase in the number of staff calling in sick can occur and does occur at times in the hospital setting and may have nothing to do with bioterrorism.

The home health nurse is caring for a patient in the home who is using an electrical infusion device. While visiting the patient, the nurse smells smoke and notices an electrical fire started by this device. The nurse uses the fire extinguisher and fights the fire when (Select all that apply.) a. All occupants have left the home. b. Fire department has been called. c. Fire is confined to one room. d. An exit route is available. e. The correct extinguisher is available. f. The nurse thinks she can use the fire extinguisher.

ANS: A, B, D, E In a home setting, if the nurse is present during a fire, she first should remove all occupants and then should call the fire department by dialing 911. If the fire is small—not confined to just one room (this could be too large for the fire extinguisher), if the correct extinguisher is available, and if the nurse knows (not thinks) that she can use it, the nurse may attempt to extinguish the fire. Utilize PASS (Pull the pin, Aim low, Squeeze the handles, Sweep area from side to side) to activate the extinguisher.

The nurse is caring for a patient who has had a recent stroke and is paralyzed on the left side. The patient has no respiratory or cardiac issues but cannot walk. The patient cannot button a shirt and cannot feed self due to being left-handed and becomes frustrated very easily. The patient has been eating very little and has lost 2 lbs. The patient asks the nurse, "How can I go home like this? I'm not getting better." Which health care team members will the nurse need to consult? (Select all that apply.) a.Dietitian b.Physical therapist c.Respiratory therapist d.Cardiac rehabilitation therapist e.Occupational therapist f.Psychologist

ANS: A, B, E, F Physical therapists are a resource for planning ROM or strengthening exercises, and occupational therapists are a resource for planning ADLs that patients need to modify or relearn. Because of the loss of 2 lbs and eating very little, a dietitian will also be helpful. Referral to a mental health advanced practice nurse, a licensed social worker, or a physiologist to assist with coping or other psychosocial issues is also wise. Because the patient exhibits good cardiac and respiratory function, respiratory therapy and cardiac rehabilitation probably are not needed at this time.

Which of the following interventions are priorities in a plan of care for a patient who had a stroke 30 days ago and is now in home care rehabilitation? (Select all that apply.) A. Promoting rest and sleep B. Promoting a diet rich in protein C. Promoting exercise and ambulation D. Assisting the patient with ADLs E. Limiting visitors and social contacts

ANS: A, C It is important to promote independence in ADLs early in the plan of care to increase independence in general. Promoting rest and sleep will promote well-being. Ambulation and exercise promote well-being and increase healing by circulating oxygen to the brain. Protein promotes healing in postsurgical patients but is not a main focus in stroke patients. Assisting the patient does not promote independence. Limiting visitors will isolate the patient, which can lead to depression.

2. A nurse is assessing several patients. Which assessment findings will cause the nurse to follow up? (Select all that apply.) a. Orthopnea b. Nonpalpable lymph nodes c. Pleural friction rub present d. Crackles in lower lung lobes e. Grade 5 muscle function level f. A 160-degree angle between nail plate and nail

ANS: A, C, D Abnormal findings will cause a nurse to follow up. Orthopnea is abnormal and indicates cardiovascular or respiratory problems. Pleural friction rub is abnormal and indicated an inflamed pleura. Crackles are adventitious breath sounds and indicate random, sudden reinflation of groups of alveoli, indicating disruptive passage of air through small airways. Lymph nodes should be nonpalpable; palpable lymph nodes are abnormal. Grade 5 muscle function is normal. A 160-degree angle between nail plate and nail is normal; a larger degree angle is abnormal and indicates clubbing.

A nurse is assessing several patients. Which assessment findings will cause the nurse to follow up? (Select all that apply.) A. Orthopnea B. Nonpalpable lymph nodes C. Pleural friction rub present D. Crackles in lower lung lobes E. Grade 5 muscle function level F. A 160-degree angle between nail plate and nail

ANS: A, C, D Abnormal findings will cause a nurse to follow up. Orthopnea is abnormal and indicates cardiovascular or respiratory problems. Pleural friction rub is abnormal and indicated an inflamed pleura. Crackles are adventitious breath sounds and indicate random, sudden reinflation of groups of alveoli, indicating disruptive passage of air through small airways. Lymph nodes should be nonpalpable; palpable lymph nodes are abnormal. Grade 5 muscle function is normal. A 160-degree angle between nail plate and nail is normal; a larger degree angle is abnormal and indicates clubbing.

The nurse is caring for a patient with impaired physical mobility. Which potential complications will the nurse monitor for in this patient? (Select all that apply.) a.Footdrop b.Somnolence c.Hypostatic pneumonia d.Impaired skin integrity e.Increased socialization

ANS: A, C, D Immobility leads to complications such as hypostatic pneumonia. Other possible complications include footdrop and impaired skin integrity. Interruptions in the sleep-wake cycle and social isolation are more common complications than somnolence or increased socialization.

The nurse is caring for an older adult who presents to the clinic after a fall. The nurse reviews fall prevention in the home. Which of the following should the patient avoid? (Select all that apply.) a. Watering outdoor plants with a nozzle and hose b. Purchasing light bulbs with strength greater than 60 watts c. Missing yearly eye examinations d. Using bathtubs without safety strips e. Unsecured rugs throughout the home f. Walking to the mailbox in the summer

ANS: A, C, D, E Unsecured rugs, using a hose to water plants, missing yearly eye examinations, and using tubs without safety strips are all items the patient should avoid to help in the prevention of falls in the home. Exercise is beneficial and increases strength, which helps with the prevention of falls. It is important that the home is well lit, so encourage the purchase of bulbs with strength of 60 watts or higher for the home.

1. A nurse is assessing a patient's cranial nerve IX. Which items does the nurse gather before conducting the assessment? (Select all that apply.) a. Vial of sugar b. Snellen chart c. Tongue blade d. Ophthalmoscope e. Lemon applicator

ANS: A, C, E Cranial nerve IX is the glossopharyngeal, which controls taste and ability to swallow. The nurse asks the patient to identify sour (lemon) or sweet (sugar) tastes on the back of the tongue and uses a tongue blade to elicit a gag reflex. Ophthalmoscopes are used for vision. A Snellen chart is used to test cranial nerve II (optic).

A nurse is assessing a patient's cranial nerve IX. Which items does the nurse gather before conducting the assessment? (Select all that apply.) A. Vial of sugar B. Snellen chart C. Tongue blade D. Ophthalmoscope E. Lemon applicator

ANS: A, C, E Cranial nerve IX is the glossopharyngeal, which controls taste and ability to swallow. The nurse asks the patient to identify sour (lemon) or sweet (sugar) tastes on the back of the tongue and uses a tongue blade to elicit a gag reflex. Ophthalmoscopes are used for vision. A Snellen chart is used to test cranial nerve II (optic).

Upon assessment a nurse discovers that a patient has erythema. Which actions will the nurse take? (Select all that apply.) a.Consult a dietitian. b.Increase fiber in the diet. c.Place on chest physiotherapy. d.Increase frequency of turning. e.Place on pressure-relieving mattress.

ANS: A, D, E If skin shows areas of erythema and breakdown, increase the frequency of turning and repositioning; place the turning schedule above the patient's bed; implement other activities per agency skin care policy or protocol (e.g., assess more frequently, consult dietitian, place patient on pressure-relieving mattress). Increased fiber will help constipation. Chest physiotherapy is for respiratory complications.

The nurse is assessing a patient with a mobility dysfunction and wants to gain insight into the patient's functional ability. What question would be the most appropriate? a. "Are you able to shop for yourself?" b. "Do you use a cane, walker, or wheelchair to ambulate?" c. "Do you know what today's date is?" d. "Were you sad or depressed more than once in the last 3 days?"

ANS: B "Do you use a cane, walker, or wheelchair to ambulate?" will assist the nurse in determining the patient's ability to perform self-care activities. A nutritional health risk assessment is not the functional assessment. Knowing the date is part of a mental status exam. Assessing sadness is a question to ask in the depression screening.

Which goal is most appropriate for a patient who has had a total hip replacement? a.The patient will ambulate briskly on the treadmill by the time of discharge. b.The patient will walk 100 feet using a walker by the time of discharge. c.The nurse will assist the patient to ambulate in the hall 2 times a day. d.The patient will ambulate by the time of discharge.

ANS: B "The patient will walk 100 feet using a walker by the time of discharge" is individualized, realistic, and measurable. "Ambulating briskly on a treadmill" is not realistic for this patient. The option that focuses on the nurse, not the patient, is not a measurable goal; this is an intervention. "The patient will ambulate by the time of discharge" is not measurable because it does not specify the distance. Even though we can see that the patient will ambulate, this does not quantify how far.

24. A teen patient is tearful and reports locating lumps in her breasts. Other history obtained is that she is currently menstruating. Physical examination reveals soft and movable cysts in both breasts that are painful to palpation. The nurse also notes that the patient's nipples are erect, but the areola is wrinkled. Which action will the nurse take after talking with the health care provider? a. Reassure patient that her symptoms are normal. b. Discuss the possibility of fibrocystic disease as the probable cause. c. Consult a breast surgeon because of the abnormal nipples and areola. d. Tell the patient that the symptoms may get worse when her period ends.

ANS: B A common benign condition of the breast is benign (fibrocystic) breast disease. This patient has symptoms of fibrocystic disease, which include bilateral lumpy, painful breasts sometimes accompanied by nipple discharge. Symptoms are more apparent during the menstrual period. When palpated, the cysts (lumps) are soft, well differentiated, and movable. Deep cysts feel hard. Although a common condition, benign breast disease is not normal; therefore, the nurse does not tell the patient that this is a normal finding. During examination of the nipples and areolae, the nipple sometimes becomes erect with wrinkling of the areola. Therefore, consulting a breast surgeon to treat her nipples and areolae is not appropriate.

Instruments such as the Functional Activities Questionnaire (FAQ) for postoperative patients who are at home, the Minimum Data Set for Nursing Facility Resident Assessment and Care Screening (MDS) for nursing home patients, the Functional Status Scale (FSS) for children, and the Edmonton Functional Assessment Tool for cancer patients are used to assess activities of daily living (ADLs). The nurse needs to remember that a disadvantage of these instruments includes: a. the measurement of efficacy and reliability of the instruments used to assess activities of daily living (ADLs). b. the variations in assessments and responses may be subjective because of self-reporting of functional activities. c. the instruments do not show a true measure of ability because of a lack of interactivity during the assessments. d. the information contained in the instruments is insufficient to make a determination about functional status in these populations.

ANS: B A disadvantage of many of the ADLs and instrumental activities of daily living (IADLs) instruments is the self-reporting of functional activities. Efficacy and reliability are not measured when assessing ADLs and IADLs. Interaction with the patient is necessary to complete the ADL and IADL assessments. The FAQ and FSS are comprehensive tools that can help the nurse determine functional status.

The nurse is caring for a patient who is immobile and needs to be turned every 2 hours. The patient has poor lower extremity circulation, and the nurse is concerned about irritation of the patient's toes. Which device will the nurse use? a.Hand rolls b.A foot cradle c.A trapeze bar d.A trochanter roll

ANS: B A foot cradle may be used in patients with poor peripheral circulation as a means of reducing pressure on the tips of a patient's toes. A trochanter roll prevents external rotation of the hips when the patient is in a supine position. Hand rolls maintain the thumb in slight adduction and in opposition to the fingers. The trapeze bar is a triangular device that hangs down from a securely fastened overhead bar that is attached to the bedframe. It allows the patient to pull with the upper extremities to raise the trunk off the bed, to assist in transfer from bed to wheelchair, or to perform upper arm exercises.

21. A teen female patient reports intermittent abdominal pain for 12 hours. No dysuria is present. Which action will the nurse take when performing an abdominal assessment? a. Assess the area that is most tender first. b. Ask the patient about the color of her stools. c. Recommend that the patient take more laxatives. d. Avoid sexual references such as possible pregnancy.

ANS: B Abdominal pain can be related to bowels. If stools are black or tarry (melena), this may indicate gastrointestinal alteration. The nurse should caution patients about the dangers of excessive use of laxatives or enemas. There is not enough information about the abdominal pain to recommend laxatives. Determine if the patient is pregnant, and note her last menstrual period. Pregnancy causes changes in abdominal shape and contour. Assess painful areas last to minimize discomfort and anxiety.

The nurse is assessing body alignment for a patient who is immobilized. Which patient position will the nurse use? a.Supine position b.Lateral position c.Lateral position with positioning supports d.Supine position with no pillow under the patient's head

ANS: B Assess body alignment for a patient who is immobilized or bedridden with the patient in the lateral position, not supine. Remove all positioning supports from the bed except for the pillow under the head, and support the body with an adequate mattress.

The patient is admitted to a skilled care unit for rehabilitation after the surgical procedure of fixation of a fractured left hip. The patient's nursing diagnosis is Impaired physical mobility related to musculoskeletal impairment from surgery and pain with movement. The patient is able to use a walker but needs assistance ambulating and transferring from the bed to the chair. Which nursing intervention is most appropriate for this patient? a.Obtain assistance and physically transfer the patient to the chair. b.Assist with ambulation and measure how far the patient walks. c.Give pain medication after ambulation so the patient will have a clear mind. d.Bring the patient to the cafeteria for group instruction on ambulation.

ANS: B Assist with walking and measure how far the patient walks to quantify progress. The nurse should allow the patient to do as much for self as possible. Therefore, the nurse should observe the patient transferring from the bed to the chair using the walker and should provide assistance as needed. The patient should be encouraged to use adequate pain medication to decrease the effects of pain and to increase mobility. The patient should be instructed on safe transfer and ambulation techniques in an environment with few distractions, not in the cafeteria.

The nurse is assessing the patient for respiratory complications of immobility. Which action will the nurse take when assessing the respiratory system? a.Inspect chest wall movements primarily during the expiratory cycle. b.Auscultate the entire lung region to assess lung sounds. c.Focus auscultation on the upper lung fields. d.Assess the patient at least every 4 hours.

ANS: B Auscultate the entire lung region to identify diminished breath sounds, crackles, or wheezes. Perform a respiratory assessment at least every 2 hours for patients with restricted activity. Inspect chest wall movements during the full inspiratory-expiratory cycle. Focus auscultation on the dependent lung fields because pulmonary secretions tend to collect in these lower regions.

The nurse is evaluating the body alignment of a patient in the sitting position. Which observation by the nurse will indicate a normal finding? a.The edge of the seat is in contact with the popliteal space. b.Both feet are supported on the floor with ankles flexed. c.The body weight is directly on the buttocks only. d.The arms hang comfortably at the sides.

ANS: B Both feet are supported on the floor, and the ankles are comfortably flexed. Body weight is evenly distributed on the buttocks and thighs. A 1- to 2-inch space is maintained between the edge of the seat and the popliteal space on the posterior surface of the knee to ensure that no pressure is placed on the popliteal artery or nerve. The patient's forearms are supported on the armrest, in the lap, or on a table in front of the chair.

A nurse is caring for a patient with osteoporosis and lactose intolerance. What will the nurse do? a.Encourage dairy products. b.Monitor intake of vitamin D. c.Increase intake of caffeinated drinks. d.Try to do as much as possible for the patient.

ANS: B Encourage patients at risk to be screened for osteoporosis and assess their diets for calcium and vitamin D intake. Patients who have lactose intolerance need dietary teaching about alternative sources of calcium. Caffeine should be decreased. The goal of the patient with osteoporosis is to maintain independence with ADLs. Assistive ambulatory devices, adaptive clothing, and safety bars help the patient maintain independence.

A nurse working in the pediatric intensive care unit (PICU) is planning care for a pediatric client who is being admitted with a partial-thickness thermal burn. What is true regarding this type of burn? a. Partial-thickness burns are deeper than superficial burns but still involve the epidermis only. b. A superficial partial-thickness burn extends from the skin's surface into the papillary layer of the dermis. c. A deep partial-thickness burn is often bright red and has a moist, glistening appearance with blister formation. d. A superficial partial-thickness burn is less painful than a deep partial-thickness burn.

ANS: B Explanation: A superficial partial-thickness burn extends from the skin's surface into the papillary layer of the dermis. Partial-thickness burns are deeper than superficial burns, extending from the epidermis into the dermis layer as well. A superficial partial-thickness burn is often bright red and has a moist, glistening appearance with blister formation. A deep partial-thickness burn is less painful than a superficial partial-thickness burn because sensation is decreased at the site.

25. A nurse is performing a mental status examination and asks an adult patient what the statement "Don't cry over spilled milk" means. Which area is the nurse assessing? a. Long-term memory b. Abstract thinking c. Recent memory d. Knowledge

ANS: B For an individual to explain common phrases such as "A stitch in time saves nine" or "Don't cry over spilled milk" requires a higher level of intellectual function or abstract thinking. Knowledge-based assessment is factual. Assess knowledge by asking how much the patient knows about the illness or the reason for seeking health care. To assess past (long-term) memory, ask the patient to recall the maiden name of the patient's mother, a birthday, or a special date in history. It is best to ask open-ended questions rather than simple yes/no questions. Patients demonstrate immediate recall (recent memory) by repeating a series of numbers in the order in which they are presented or in reverse order.

A nurse is evaluating care of an immobilized patient. Which action will the nurse take? a.Focus on whether the interdisciplinary team is satisfied with the care. b.Compare the patient's actual outcomes with the outcomes in the care plan. c.Involve primarily the patient's family and health care team to determine goal achievement. d.Use objective data solely in determining whether interventions have been successful.

ANS: B From your perspective as the nurse, you are to evaluate outcomes and response to nursing care and compare the patient's actual outcomes with the outcomes selected during planning. Ask if the patient's expectations (subjective data) of care are being met, and use objective data to determine the success of interventions. Just as it was important to include the patient during the assessment and planning phase of the care plan, it is essential to have the patient's evaluation of the plan of care, not just the patient's family and health care team.

32. The nurse is assessing an adult patient's patellar reflex. Which finding will the nurse record as normal? a. 1+ b. 2+ c. 3+ d. 4+

ANS: B Grade reflexes as follows: 0: No response; 1+: Sluggish or diminished; 2+: Active or expected response; 3+: More brisk than expected, slightly hyperactive; and 4+: Brisk and hyperactive with intermittent or transient clonus.

The patient is immobilized after undergoing hip replacement surgery. Which finding will alert the nurse to monitor for hemorrhage in this patient? a.Thick, tenacious pulmonary secretions b.Low-molecular-weight heparin doses c.SCDs wrapped around the legs d.Elastic stockings (TED hose)

ANS: B Heparin and low-molecular-weight heparin are the most widely used drugs in the prophylaxis of deep vein thrombosis. Because bleeding is a potential side effect of these medications, continually assess the patient for signs of bleeding. Pulmonary secretions that become thick and tenacious are difficult to remove and are a sign of inadequate hydration or developing pneumonia but not of bleeding. SCDs consist of sleeves or stockings made of fabric or plastic that are wrapped around the leg and are secured with Velcro. They decrease venous stasis by increasing venous return through the deep veins of the legs. They do not usually cause bleeding. Elastic stockings also aid in maintaining external pressure on the muscles of the lower extremities and in promoting venous return. They do not usually cause bleeding.

A nurse is assessing pressure points in a patient placed in the Sims' position. Which areas will the nurse observe? a.Chin, elbow, hips b.Ileum, clavicle, knees c.Shoulder, anterior iliac spine, ankles d.Occipital region of the head, coccyx, heels

ANS: B In the Sims' position pressure points include the ileum, humerus, clavicle, knees, and ankles. The lateral position pressure points include the ear, shoulder, anterior iliac spine, and ankles. The prone position pressure points include the chin, elbows, female breasts, hips, knees, and toes. Supine position pressure points include the occipital region of the head, vertebrae, coccyx, elbows, and heels.

14. The nurse is examining a female with vaginal discharge. Which position will the nurse place the patient for proper examination? a. Sitting b. Lithotomy c. Knee-chest d. Dorsal recumbent

ANS: B Lithotomy is the position for examination of female genitalia. The lithotomy position provides for the maximum exposure of genitalia and allows the insertion of a vaginal speculum. Sitting does not allow adequate access for speculum insertion and is better used to visualize upper body parts. Dorsal recumbent is used to examine the head and neck, anterior thorax and lungs, breasts, axillae, heart, and abdomen. Knee-chest provides maximal exposure of the rectal area but is embarrassing and uncomfortable.

The nurse is assessing an immobile patient for deep vein thromboses (DVTs). Which action will the nurse take? a.Remove elastic stockings every 4 hours. b.Measure the calf circumference of both legs. c.Lightly rub the lower leg for redness and tenderness. d.Dorsiflex the foot while assessing for patient discomfort.

ANS: B Measure bilateral calf circumference and record it daily as an assessment for DVT. Unilateral increases in calf circumference are an early indication of thrombosis. Homan's sign, or calf pain on dorsiflexion of the foot, is no longer a reliable indicator in assessing for DVT, and it is present in other conditions. Remove the patient's elastic stockings and/or sequential compression devices (SCDs) every 8 hours, and observe the calves for redness, warmth, and tenderness. Instruct the family, patient, and all health care personnel not to massage the area because of the danger of dislodging the thrombus.

A nurse is a preceptor for a nurse who just graduated from nursing school. When caring for a patient, the new graduate nurse begins to explain to the patient the purpose of completing a physical assessment. Which statement made by the new graduate nurse requires the preceptor to intervene? a."I will use the information from my assessment to figure out if your antihypertensive medication is working effectively." b."Nursing assessment data are used only to provide information about the effectiveness of your medical care." c."Nurses use data from their patient's physical assessment to determine a patient's educational needs." d."Information gained from physical assessment helps nurses better understand their patients' emotional needs."

ANS: B Nursing assessment data are used to evaluate the effectiveness of all aspects of a patient's care, not just the patient's medical care. Assessment data help to evaluate the effectiveness of medications and to determine a patient's health care needs, including the need for patient education. Nurses also use assessment data to identify patients' psychosocial and cultural needs.

37. An older-adult patient is taking aminoglycoside for a severe infection. Which assessment is the priority? a. Eyes b. Ears c. Skin d. Reflexes

ANS: B Older adults are especially at risk for hearing loss caused by ototoxicity (injury to auditory nerve) resulting from high maintenance doses of antibiotics (e.g., aminoglycosides). While eyes and skin are important, they are not the priority. Reflexes are expected to be diminished in older adults.

The nurse is caring for a patient who has had a stroke causing total paralysis of the right side. To help maintain joint function and minimize the disability from contractures, passive ROM will be initiated. When should the nurse begin this therapy? a.After the acute phase of the disease has passed b.as soon as the ability to move is lost c.Once the patient enters the rehab unit d.When the patient requests it

ANS: B Passive ROM exercises should begin as soon as the patient's ability to move the extremity or joint is lost. The nurse should not wait for the acute phase to end. It may be some time before the patient enters the rehab unit or the patient requests it, and contractures could form by then.

A patient in the outpatient setting was diagnosed with atopic dermatitis. What interventions will the plan of care focus primarily on? a. Decreasing pain b. Decreasing pruritus c. Preventing infection d. Promoting drying of lesions

ANS: B Pruritus is the major manifestation of atopic dermatitis and causes the greatest morbidity. The urge to scratch may be mild and self-limiting, or it may be intense, leading to severely excoriated lesions, infection, and scarring.

A nurse reviews an immobilized patient's laboratory results and discovers hypercalcemia. Which condition will the nurse monitor for most closely in this patient? a.Hypostatic pneumonia b.Renal calculi c.Pressure ulcers d.Thrombus formation

ANS: B Renal calculi are calcium stones that lodge in the renal pelvis or pass through the ureters. Immobilized patients are at risk for calculi because they frequently have hypercalcemia. Hypercalcemia does not lead to hypostatic pneumonia, pressure ulcers, or thrombus formation. Immobility is one cause of hypostatic pneumonia, which is inflammation of the lung from stasis or pooling of secretions. A pressure ulcer is an impairment of the skin that results from prolonged ischemia (decreased blood supply) within tissues. A thrombus is an accumulation of platelets, fibrin, clotting factors, and cellular elements of the blood attached to the interior wall of a vein or artery, which sometimes occludes the lumen of the vessel.

Which patient will cause the nurse to select a nursing diagnosis of Impaired physical mobility for a care plan? a.A patient who is completely immobile b.A patient who is not completely immobile c.A patient at risk for single-system involvement d.A patient who is at risk for multisystem problems

ANS: B The diagnosis of Impaired physical mobility applies to the patient who has some limitation but is not completely immobile. The diagnosis of Risk for disuse syndrome applies to the patient who is immobile and at risk for multisystem problems because of inactivity. Beyond these diagnoses, the list of potential diagnoses is extensive because immobility affects multiple body systems.

18. An older-adult patient is being seen for chronic entropion. Which condition will the nurse assess for in this patient? a. Ptosis b. Infection c. Borborygmi d. Exophthalmos

ANS: B The diagnosis of entropion can lead to lashes of the lids irritating the conjunctiva and cornea. Irritation can lead to infection. Exophthalmos is a bulging of the eyes and usually indicates hyperthyroidism. An abnormal drooping of the lid over the pupil is called ptosis. In the older adult, ptosis results from a loss of elasticity that accompanies aging. Hyperactive sounds are loud, "growling" sounds called borborygmi, which indicate increased GI motility.

A nurse is preparing to reposition a patient. Which task can the nurse delegate to the nursing assistive personnel? a.Determining the level of comfort b.Changing the patient's position c.Identifying immobility hazards d.Assessing circulation

ANS: B The skill of moving and positioning patients in bed can be delegated to nursing assistive personnel (NAP). The nurse is responsible for assessing the patient's level of comfort and for any hazards of immobility and assessing circulation.

29. While assessing the skin of an 82-year-old patient, a nurse discovers nonpainful, ruby red papules on the patient's trunk. What is the nurse's next action? a. Explain that the patient has basal cell carcinoma and should watch for spread. b. Document cherry angiomas as a normal older adult skin finding. c. Tell the patient that this is a benign squamous cell carcinoma. d. Record the presence of petechiae.

ANS: B The skin is normally free of lesions, except for common freckles or age-related changes such as skin tags, senile keratosis (thickening of skin), cherry angiomas (ruby red papules), and atrophic warts. Basal cell carcinoma is most common in sun-exposed areas and frequently occurs in a background of sun-damaged skin; it almost never spreads to other parts of the body. Squamous cell carcinoma is more serious than basal cell and develops on the outer layers of sun-exposed skin; these cells may travel to lymph nodes and throughout the body. Report abnormal lesions to the health care provider for further examination. Petechiae are nonblanching, pinpoint-size, red or purple spots on the skin caused by small hemorrhages in the skin layers.

A nurse is educating a 21-year-old lifeguard about the risk of skin cancer and the need to wear sunscreen. Which statement by the patient indicates that the need for further teaching? a. "I wear a hat and sit under the umbrella when not in the water." b. "I don't bother with sunscreen on overcast days." c."I use a sunscreen with the highest SPF number." d. "I wear a UV shirt and limit exposure to the sun by covering up."

ANS: B The sun's rays are as damaging to skin on cloudy, hazy days as on sunny days. The other options will all prevent skin cancer.

The patient has been in bed for several days and needs to be ambulated. Which action will the nurse take first? a.Maintain a narrow base of support. b.Dangle the patient at the bedside. c.Encourage isometric exercises. d.Suggest a high-calcium diet.

ANS: B To prevent injury, nurses implement interventions that reduce or eliminate the effects of orthostatic hypotension. Mobilize the patient as soon as the physical condition allows, even if this only involves dangling at the bedside or moving to a chair. A wide base of support increases balance. Isometric exercises (i.e., activities that involve muscle tension without muscle shortening) have no beneficial effect on preventing orthostatic hypotension, but they improve activity tolerance. A high-calcium diet can help with osteoporosis but can be detrimental in an immobile patient.

The nurse is preparing to lift a patient. Which action will the nurse take first? a.Position a drawsheet under the patient. b.Assess weight and determine assistance needs. c.Delegate the task to a nursing assistive personnel. d.Attempt to manually lift the patient alone before asking for assistance.

ANS: B When lifting, assess the weight you will lift, and determine the assistance you will need. The nurse has to assess before positioning a drawsheet or delegating the task. Manual lifting is the last resort, and it is used when the task at hand does not involve lifting most or all of the patient's weight; most facilities have a no-lift policy.

39. The nurse anticipates a suprapubic catheter for which patient? a. A patient with recent prostatectomy b. A patient with a urethral stricture c. A patient with an appendectomy d. A patient with menopause

ANS: B A patient with a urethral stricture is most likely to have a suprapubic catheter. Suprapubic catheters are placed when there is blockage of the urethra (e.g., enlarged prostate, urethral stricture, after urological surgery). A patient with a recent prostatectomy indicates the enlarged prostate was removed and would not need a suprapubic catheter; however, continuous bladder irrigation may be needed. Appendectomies and menopause do not require a suprapubic catheter.

The nurse identifies that a patient has received Mylanta (simethicone) instead of the prescribed Pepto-Bismol (bismuth subsalicylate) for the problem of indigestion. The nurse's next intervention is to a. Do nothing, no harm has occurred. b. Assess and monitor the patient. c. Notify the physician, treat and document. d. Complete an incident report.

ANS: B After providing an incorrect medication, assessing and monitoring the patient to determine the effects of the medication is the first step. Notifying the physician and providing treatment would be the best next step. After the patient has stabilized, completing an incident report would be the last step in the process.

6. The nurse, upon reviewing the history, discovers the patient has dysuria. Which assessment finding is consistent with dysuria? a. Blood in the urine b. Burning upon urination c. Immediate, strong desire to void d. Awakes from sleep due to urge to void

ANS: B Dysuria is burning or pain with urination. Hematuria is blood in the urine. Urgency is an immediate and strong desire to void that is not easily deferred. Nocturia is awakening form sleep due to urge to void.

A homeless adult patient presents to the emergency department. The nurse obtains the following vital signs: temperature 94.8° F, blood pressure 100/56, apical pulse 56, respiratory rate 12. Which of the vital signs should be addressed immediately? a. Respiratory rate b. Temperature c. Apical pulse d. Blood pressure

ANS: B Hypothermia is defined as a core body temperature of 95° F or below. Homeless individuals are more at risk for hypothermia owing to exposure to the elements.

The nurse has been called to a hospital room where a patient is using a hair dryer from home. The patient has received an electrical shock from the dryer. The patient is unconscious and is not breathing. What is the best next step? a. Ask the family to leave the room. b. Check for a pulse. c. Begin compressions. d. Defibrillate the patient.

ANS: B In this scenario, the patient is in a hospital setting, and it has been determined that the patient is not conscious and is not breathing. The next step is to check the pulse. An electrical shock can interfere with the heart's normal electrical impulses and can cause arrhythmias. Checking the pulse helps to determine the need for cardiopulmonary resuscitation (CPR) and defibrillation.

The patient presents to the clinic with a family member. The family member states that the patient has been wandering around the house and mumbling. What is the first assessment the nurse should do? a. Ask the patient why she has been wandering around the house. b. Introduce self and ask the patient her name. c. Take the patient's blood pressure, pulse, temperature, and respiratory rate. d. Immediately do a complete head-to-toe neurologic assessment.

ANS: B Introduce self and engage the patient by asking her name to assess orientation; ask the patient why she is visiting the clinic today. Continue the assessment with vital signs and a complete workup, including a neurologic assessment.

An elderly patient presents to the hospital with a history of falls, confusion, and stroke. The nurse determines that the patient is at high risk for falls. Which of the following interventions is most appropriate for the nurse to take? a. Place the patient in restraints. b. Lock beds and wheelchairs when transferring. c. Place a bath mat outside the tub. d. Silence fall alert alarm upon request of family

ANS: B Locking the bed and wheelchairs when transferring will help to prevent these pieces of equipment from moving during transfer and will assist in the prevention of falls. Patients are not automatically placed in restraints. The restraint process consists of many steps, including thorough assessment and exhausting of alternatives. All mats and rugs should be secured to help prevent falls. Silencing alarms upon the request of family is not appropriate and could contribute to an unsafe environment.

25. The nurse is preparing to test a patient for postvoid residual with a bladder scan. Which action will the nurse take? a. Measure bladder before the patient voids. b. Measure bladder within 10 minutes after the patient voids. c. Measure bladder with head of bed raised to 60 degrees. d. Measure bladder with head of bed raised to 90 degrees.

ANS: B Measurement should be within 10 minutes of voiding. It is a postvoid so the measurement is after the patient voids and the urine volume is recorded. Patient is supine with head slightly elevated.

24. Which statement by the patient about an upcoming contrast computed tomography (CT) scan indicates a need for further teaching? a. "I will follow the food and drink restrictions as directed before the test is scheduled." b. "I will be anesthetized so that I lie perfectly still during the procedure." c. "I will complete my bowel prep program the night before the scan." d. "I will be drinking a lot of fluid after the test is over."

ANS: B Patients are not put under anesthesia for a CT scan; instead, the nurse should educate patients about the need to lie perfectly still and about possible methods of overcoming feelings of claustrophobia. The other options are correct and require no further teaching. Patients need to be assessed for an allergy to shellfish if receiving contrast for the CT scan. Bowel cleansing is often performed before CT scan. Another area to address is food and fluid restriction up to 4 hours prior to the test. After the procedure, encourage fluids to promote dye excretion.

A patient with an intravenous infusion requests a new gown after bathing. Which of the following actions is most appropriate? a. Disconnect the intravenous tubing, thread the end through the sleeve of the old gown and through the sleeve of the new gown, and reconnect. b. Thread the intravenous bag and tubing through the sleeve of the old gown and through the sleeve of the new gown without disconnecting. c. Inform the patient that a new gown is not an option while receiving an intravenous infusion in the hospital. d. Call the charge nurse for assistance because linen use is monitored and this is not a common procedure.

ANS: B Procedure-related accidents such as contamination of sterile items can occur in the health care setting. Keeping the intravenous tubing intact without breaks in the system is imperative to decrease the risk of infection while changing a patient's gown and satisfying the patient's request.

The nurse has placed a patient on high-risk alert for falls. Which of the following observations by the nurse would indicate that the patient has an understanding of this alert? a. The patient removes the high alert armband to bathe. b. The patient wears the red nonslip footwear. c. The call light is kept on the bedside table. d. The patient insists on taking a "water" pill on home schedule in the evening.

ANS: B Red nonslip footwear helps to grip the floor and decreases the chance of falling. The communication armband should stay in place and should not be removed, so that all members of the interdisciplinary team have the information about the high risk for falls. Call lights should be kept within reach of the patient. Taking diuretics early in the day assists with decreasing the number of bathroom trips at night—the time when falls are most frequent

The nurse is caring for a hospitalized patient. Which of the following behaviors alerts the nurse to consider the need for restraint? a. The patient refuses to call for help to go to the bathroom. b. The patient continues to remove the nasogastric tube. c. The patient gets confused regarding the time at night. d. The patient does not sleep and continues to ask for items.

ANS: B Restraints are utilized only when alternatives have been exhausted, the patient continues a behavior that can be harmful to himself or others, and the restraint is clinically justified. In this circumstance, continuing to remove a needed nasogastric tube would meet these criteria. Refusing to call for help, although unsafe, is not a reason for restraint. Getting confused at night regarding the time or not sleeping and bothering the staff to ask for items is not a reason for restraint.

20. The patient is taking phenazopyridine. When assessing the urine, what will the nurse expect? a. Red color b. Orange color c. Dark amber color d. Intense yellow color

ANS: B Some drugs change the color of urine (e.g., phenazopyridine—orange, riboflavin—intense yellow). Eating beets, rhubarb, and blackberries causes red urine. Dark amber urine is the result of high concentrations of bilirubin in patients with liver disease.

1. A nurse is teaching a patient about the urinary system. In which order will the nurse present the structures, following the flow of urine? a. Kidney, urethra, bladder, ureters b. Kidney, ureters, bladder, urethra c. Bladder, kidney, ureters, urethra d. Bladder, kidney, urethra, ureters

ANS: B The flow of urine follows these structures: kidney, ureters, bladder, and urethra.

36. A nurse is providing care to a group of patients. Which patient will the nurse see first? a. A patient who is dribbling small amounts on the way to the bathroom and has a diagnosis of urge incontinence b. A patient with reflex incontinence with elevated blood pressure and pulse rate c. A patient with an indwelling catheter that has stool on the catheter tubing d. A patient who has just voided and needs a postvoid residual test

ANS: B The nurse should see the patient with reflex incontinence first. Patients with reflex incontinence are at risk for developing autonomic dysreflexia, a life-threatening condition that causes severe elevation of blood pressure and pulse rate and diaphoresis. This is a medical emergency requiring immediate intervention; notify the health care provider immediately. A patient with urge incontinence will dribble, and this is expected. While a patient with a catheter and stool on the tubing does need to be cleaned, it is not life threatening. The nurse has 10 minutes before checking on the patient who has a postvoid residual test.

The patient applies sequential compression devices after going to the bathroom. The nurse checks the patient's application of the devices and finds that they have been put on upside down. Which of the following nursing diagnoses will the nurse add to the patient's plan of care? a. Risk for poisoning b. Deficient knowledge c. Risk for imbalanced body temperature d. Risk for suffocation

ANS: B The patient needs to understand the purpose of the compression devices and that proper application is needed for them to be effective. The patient has a knowledge need and requires instruction regarding the device and its purpose and procedure. The nurse will intervene by teaching the patient about the sequential compression device and instructing the patient to call for assistance when getting up to go to the bathroom in the future, so that the nurse may assist with removal and proper reapplication. No data support a risk for poisoning, imbalanced body temperature, or suffocation.

27. A female patient is having difficulty voiding in a bedpan but states that her bladder feels full. To stimulate micturition, which nursing intervention should the nurse try first? a. Exiting the room and informing the patient that the nurse will return in 30 minutes to check on the patient's progress. b. Utilizing the power of suggestion by turning on the faucet and letting the water run. c. Obtaining an order for a Foley catheter. d. Administering diuretic medication.

ANS: B To stimulate micturition, the nurse should attempt noninvasive procedures first. Running warm water or stroking the inner aspect of the upper thigh promotes sensory perception that leads to urination. A patient should not be left alone on a bedpan for 30 minutes because this could cause skin breakdown. Catheterization places the patient at increased risk of infection and should not be the first intervention attempted. Diuretics are useful if the patient is not producing urine, but they do not stimulate micturition.

7. An 86-year-old patient is experiencing uncontrollable leakage of urine with a strong desire to void and even leaks on the way to the toilet. Which priority nursing diagnosis will the nurse include in the patient's plan of care? a. Functional urinary incontinence b. Urge urinary incontinence c. Impaired skin integrity d. Urinary retention

ANS: B Urge urinary incontinence is the leakage of urine associated with a strong urge to void. Patients leak urine on the way to or at the toilet and rush or hurry to the toilet. Urinary retention is the inability to empty the bladder. Functional urinary incontinence is incontinence due to causes outside the urinary tract, such as mobility or cognitive impairments. While Impaired skin integritycan occur, it is not the priority at this time, and there is no data to support this diagnosis.

12. While receiving a shift report on a patient, the nurse is informed that the patient has urinary incontinence. Upon assessment, which finding will the nurse expect? a. An indwelling Foley catheter b. Reddened irritated skin on buttocks c. Tiny blood clots in the patient's urine d. Foul-smelling discharge indicative of infection

ANS: B Urinary incontinence is uncontrolled urinary elimination; if the urine has prolonged contact with the skin, skin breakdown can occur. An indwelling Foley catheter is a solution for urine retention. Blood clots and foul-smelling discharge are often signs of infection.

The nurse is completing an admission history on a new home health patient. The patient has been experiencing seizures as the result of a recent brain injury. The nurse diagnoses risk for injury with a goal of keeping the patient safe in the event of a seizure. Which interventions should the nurse utilize for this patient? (Select all that apply.) a. Teach the family how to insert an oral airway during the seizure. b. Assess the home for items that could harm the patient during a seizure. c. Provide information on how to obtain a Medical Alert bracelet. d. Teach the patient to communicate to the caregiver plans for bathing. e. Discuss with family steps to take if the seizure does not discontinue. f. Demonstrate how to restrain the patient in the event of a seizure.

ANS: B, C, D, E Assessment of the home for safety, providing information on Medical Alert bracelets, teaching the patient to communicate before bathing, and discussing steps to take with status epilepticus are important interventions for the patient who is having seizures. Inserting an airway may harm the patient by forcing the object into the mouth or by biting down on a hard object. Never restrain a patient who is having a seizure, but protect the patient from hitting his body on objects around him to prevent traumatic injury

The nurse is caring for a patient in restraints. Which of the following pieces of information about restraints requires nursing documentation in the medical record? (Select all that apply.) a. The patient states that her gown is soiled and needs changing. b. Attempts to distract the patient with television are unsuccessful. c. The patient has been placed in bilateral wrist restraints at 0815. d. One family member has gone to lunch. e. Bilateral radial pulses present, 2+, hands warm to touch f. Released from restraints, active range-of-motion exercises complete

ANS: B, C, E, F Attempts at alternatives are documented in the medical record, as are type of restraint and time restrained. Assessments related to oxygenation, orientation, skin integrity, circulation, and position are documented, along with release from restraints and patient response. Comments about hygiene or the activities of one family member are not necessarily required in nursing documentation of restraints.

The nurse is assessing a patient's ability to perform instrumental activities of daily living (IADLs). Which of the following activities are considered in the IADLs assessment? (Select all that apply.) a. Feeding oneself b. Preparing a meal c. Balancing a checkbook d. Walking e. Toileting f. Grocery shopping

ANS: B, C, F IADLs include shopping, meal preparation, housekeeping, doing laundry, managing finances, taking medications, and using transportation. The other activities listed are activities of daily living (ADLs) related to self-care. IADLs are more complex skills that are essential to living in a community.

A 65-year-old female patient has been admitted to the medical/surgical unit. The nurse is assessing the patient's risk for falls so that falls prevention can be implemented if necessary. Select all the risk factors that apply from this patient's history and physical. (Select all that apply.) a. Being a woman b. Taking more than six medications c. Having hypertension d. Having cataractsE. Muscle strength 3/5 bilaterallyF. Incontinence

ANS: B, D, E, F Adverse effects of medications can contribute to falls. Cataracts impair vision, which is a risk factor for falls. Poor muscle strength is a risk factor for falls. Incontinence of urine or stool increases risk for falls. Men have a higher risk for falls. Hypertension itself does not contribute to falls. Dizziness does contribute to falls.

Which of the following concepts are important to utilize when evaluating orders for restraints (Select all that apply.) a. Behaviors that necessitate the use of restraint are part of the nursing plan of care. b. A physician's order is required for restraint and includes a face-to-face evaluation. c. The physician's preference for the format of the order can override agency policy. d. Orders are time limited. Restraints are not ordered prn (as needed). e. It should be specified that restraints are to be removed periodically. f. Restraint orders are time dated and signed by the physician

ANS: B, D, E, F Physicians are responsible for writing restraint orders and conducting face-to-face evaluations, as well as for putting time limits, specifying when to remove, and time dating and signing orders. Behaviors that necessitate the use of restraint not only are part of the nursing documentation but are to be included as part of the order for restraint. The physician's formatting is not a consideration for evaluating restraint orders. Formatting of restraint orders typically follows state rules and regulations, as well as regulatory agency standards.

A nurse is preparing to assess a patient for orthostatic hypotension. Which piece of equipment will the nurse obtain to assess for this condition? a.Thermometer b.Elastic stockings c.Blood pressure cuff d.Sequential compression devices

ANS: C A blood pressure cuff is needed. Orthostatic hypotension is a drop of blood pressure greater than 20 mm Hg in systolic pressure or 10 mm Hg in diastolic pressure and symptoms of dizziness, light-headedness, nausea, tachycardia, pallor, or fainting when the patient changes from the supine to standing position. A thermometer is used to assess for fever. Elastic stockings and sequential compression devices are used to prevent thrombus.

34. The nurse completed assessments on several patients. Which assessment finding will the nurse record as normal? a. Pulse strength 3 b. 1+ pitting edema c. Constricting pupils when directly illuminated d. Hyperactive bowel sounds in all four quadrants

ANS: C A normal finding is pupils constricting when directly illuminated with a penlight. A pulse strength of 3 indicates a full or increased pulse; 2 is normal. 1+ pitting edema is abnormal; there should be no edema for a normal finding. Hyperactive bowel sounds are abnormal and indicate increased GI motility; normal bowel sounds are active.

The nurse is caring for a patient with a spinal cord injury and notices that the patient's hips have a tendency to rotate externally when the patient is supine. Which device will the nurse use to help prevent injury secondary to this rotation? a.Hand rolls b.A trapeze bar c.A trochanter roll d.Hand-wrist splints

ANS: C A trochanter roll prevents external rotation of the hips when the patient is in a supine position. Hand rolls maintain the thumb in slight adduction and in opposition to the fingers. Hand-wrist splints are individually molded for the patient to maintain proper alignment of the thumb and the wrist. The trapeze bar is a triangular device that hangs down from a securely fastened overhead bar that is attached to the bedframe. It allows the patient to pull with the upper extremities to raise the trunk off the bed, to assist in transfer from bed to wheelchair, or to perform upper arm exercises.

28. The paramedics transport an adult involved in a motor vehicle accident to the emergency department. On physical examination, the patient's level of consciousness is reported as opening eyes to pain and responding with inappropriate words and flexion withdrawal to painful stimuli. Which value will the nurse report for the patient's Glasgow Coma Scale score? a. 5 b. 7 c. 9 d. 11

ANS: C According to the guidelines of the Glasgow Coma Scale, the patient has a score of 9. Opening eyes to pain is 2 points; inappropriate word use is 3 points; and flexion withdrawal is 4 points. The total for this patient is 2 + 3 + 4 = 9.

The nurse needs to move a patient up in bed using a drawsheet. The nurse has another nurse helping. In which order will the nurses perform the steps, beginning with the first one? 1. Grasp the drawsheet firmly near the patient. 2. Move the patient and drawsheet to the desired position. 3. Position one nurse at each side of the bed. 4. Place the drawsheet under the patient from shoulder to thigh. 5. Place your feet apart with a forward-backward stance. 6. Flex knees and hips and on count of three shift weight from the front to back leg. a.1, 4, 5, 6, 3, 2 b.4, 1, 3, 5, 6, 2 c.3, 4, 1, 5, 6, 2 d.5, 6, 3, 1, 4, 2

ANS: C Assisting a patient up in bed with a drawsheet (two or three nurses): (1) Place the patient supine with the head of the bed flat. A nurse stands on each side of the bed. (2) Remove the pillow from under the patient's head and shoulders and place it at the head of the bed. (3) Turn the patient side to side to place the drawsheet under the patient, extending it from shoulders to thighs. (4) Return the patient to the supine position. (5) Fanfold the drawsheet on both sides, with each nurse grasping firmly near the patient. (6) Nurses place their feet apart with a forward-backward stance. Nurses should flex knees and hips. On the count of three, nurses should shift their weight from front to back leg and move the patient and drawsheet to the desired position in the bed.

The nurse is working on an orthopedic rehabilitation unit that requires lifting and positioning of patients. Which personal injury will the nurse most likely try to prevent? a.Arm b.Hip c.Back d.Ankle

ANS: C Back injuries are often the direct result of improper lifting and bending. The most common back injury is strain on the lumbar muscle group. While arm, hip, and ankle can occur, they are not as common as back.

A nurse is assessing the skin of an immobilized patient. What will the nurse do? a.Assess the skin every 4 hours. b.Limit the amount of fluid intake. c.Use a standardized tool such as the Braden Scale. d.Have special times for inspection so as to not interrupt routine care.

ANS: C Consistently use a standardized tool, such as the Braden Scale. This identifies patients with a high risk for impaired skin integrity. Skin assessment can be as often as every hour. Limiting fluids can lead to dehydration, increasing skin breakdown. Observe the skin often during routine care.

A nurse working at a burn center is caring for a client with an electrical burn. According to the American Burn Association, how would this burn be classified? a. Minor b. Moderate c. Major d. Significant

ANS: C Explanation: According to the American Burn Association, all electrical burns are classified as major. Significant is not a classification according to the American Burn Association, and all other choices are incorrect.

27. The nurse considers several new female patients to receive additional teaching on the need for more frequent Pap test and gynecological examinations. Which assessment findings reveal the patient at highest risk for cervical cancer and having the greatest need for patient education? a. 13 years old, nonsmoker, not sexually active b. 15 years old, social smoker, celibate c. 22 years old, smokes 1 pack of cigarettes per day, has multiple sexual partners d. 50 years old, stopped smoking 30 years ago, has history of multiple pregnancies

ANS: C Females considered to be at higher risk include those who smoke, have multiple sex partners, and have a history of sexually transmitted infections. Of all the assessment findings listed, the 22-year-old smoker with multiple sexual partners has the greatest number of risk factors for cervical cancer. The other patients are at lower risk: not sexually active, celibate, and do not smoke.

11. During a sexually transmitted illness presentation to high-school students, the nurse recommends the human papillomavirus (HPV) vaccine series. Which condition is the nurse trying to prevent? a. Breast cancer b. Ovarian cancer c. Cervical cancer d. Testicular cancer

ANS: C Human papillomavirus (HPV) infection increases the person's risk for cervical cancer. HPV vaccine is recommended for females aged 11 to 12 years but can be given to females ages 12 through 26; males can also receive the vaccine. HPV is not a risk factor for breast, ovarian, and testicular cancer.

19. During a school physical examination, the nurse reviews the patient's current medical history. The nurse discovers the patient has allergies. Which assessment finding is consistent with allergies? a. Clubbing b. Yellow discharge c. Pale nasal mucosa d. Puffiness of nasal mucosa

ANS: C Pale nasal mucosa with clear discharge indicates allergy. Clubbing is due to insufficient oxygenation at the periphery resulting from conditions such as chronic emphysema and congenital heart disease; it is noted in the nails. A sinus infection results in yellowish or greenish discharge. Habitual use of intranasal cocaine and opioids causes puffiness and increased vascularity of the nasal mucosa.

17. An advanced practice nurse is preparing to assess the external genitalia of a 25-year-old American woman of Chinese descent. Which action will the nurse do first? a. Place the patient in the lithotomy position. b. Drape the patient to enhance patient comfort. c. Assess the patient's feelings about the examination. d. Ask the patient if she would like her mother to be present in the room.

ANS: C Patients who are Chinese American often believe that examination of the external genitalia is offensive. Before proceeding with the examination, the nurse first determines how the patient feels about the procedure and explains the procedure to answer any questions and to help the patient feel comfortable with the assessment. Once the patient is ready to have her external genitalia examined, the nurse places the patient in the lithotomy position and drapes the patient appropriately. Typically, nurses ask adolescents if they want a parent present during the examination. The patient in this question is 25 years old; asking if she would like her mother to be present is inappropriate.

A patient is to receive phototherapy for the treatment of psoriasis. What is the nursing priority for this patient? a. Obtaining a complete blood count (CBC) b. Protection from excessive heat c. Protection from excessive ultraviolet (UV) exposure d. Instructing the patient to take their multivitamin prior to treatment

ANS: C Protection from excessive UV exposure is important to prevent tissue damage. Protection from heat is not the most important priority for this patient. There is no need for vitamins or a CBC for patients with psoriasis.

A patient has cellulitis on the right forearm. The nurse would anticipate orders to administer medications to eradicate which organism? a. Candida albicans b. Group A beta-hemolytic streptococci c. Staphylococcus aureus d. E. Coli

ANS: C Staphylococcus aureus is the usual cause of cellulitis, although other pathogens may be responsible. A small abrasion or lesion can provide a portal for opportunistic or pathogenic infectious organisms to infect deeper tissues.

3. A nurse is preparing to perform a complete physical examination on a weak, older-adult patient with bilateral basilar pneumonia. Which position will the nurse use? a. Prone b. Sims' c. Supine d. Lateral recumbent

ANS: C Supine is the most normally relaxed position. If the patient becomes short of breath easily, raise the head of the bed. Supine position would be easiest for a weak, older-adult person during the examination. Lateral recumbent and prone positions cause respiratory difficulty for any patient with respiratory difficulties. Sims' position is used for assessment of the rectum and the vagina.

The home care nurse is trying to determine the necessary services for a 65-year-old patient who was admitted to the home care service status after left knee replacement. Which tool(s) will assist with this determination? a. Minimum Data Set (MDS) b. Functional Status Scale (FSS) c. 24-Hour Functional Ability Questionnaire (24hFAQ) d. The Edmonton Functional Assessment Tool

ANS: C The 24hFAQ assesses the postoperative patient in the home setting. The MDS is for nursing home patients. The FSS is for children. The Edmonton is for cancer patients.

An older patient has developed age spots and is concerned about skin cancer. How would the nurse instruct the patient to perform skin checks to assess for signs of skin cancer? a. "Limit the time you spend in the sun." b. "Monitor for signs of infection." c. "Monitor spots for color change." d. "Use skin creams to prevent drying."

ANS: C The ABCD method (check for asymmetry, border irregularity, color variation, and diameter) should be used to assess lesions for signs associated with cancer. Color change could be a sign of cancer and needs to be looked at by a dermatologist. Limiting time spent in the sun is a preventative measure but will not assist the patient in checking the skin or detecting skin cancer. Infection is usually not found in skin cancer. Skin creams have not been shown to prevent cancer nor would they assist in detecting skin cancer.

31. A nurse is auscultating different areas on an adult patient. Which technique should the nurse use during an assessment? a. Uses the bell to listen for lung sounds b. Uses the diaphragm to listen for bruits c. Uses the diaphragm to listen for bowel sounds d. Uses the bell to listen for high-pitched murmurs

ANS: C The bell is best for hearing low-pitched sounds such as vascular (bruits) and certain heart sounds (low-pitched murmurs), and the diaphragm is best for listening to high-pitched sounds such as bowel and lung sounds and high-pitched murmurs.

Which behavior indicates the nurse is using a team approach when caring for a patient who is experiencing alterations in mobility? a.Delegates assessment of lung sounds to nursing assistive personnel b.Becomes solely responsible for modifying activities of daily living c.Consults physical therapy for strengthening exercises in the extremities d.Involves respiratory therapy for altered breathing from severe anxiety levels

ANS: C The nurse should collaborate with other health care team members such as physical or occupational therapists when considering mobility needs. For example, physical therapists are a resource for planning ROM or strengthening exercises. Nurses often delegate some interventions to nursing assistive personnel, but assessment of lung sounds is the nurse's responsibility. Nursing assistive personnel may turn and position patients, apply elastic stockings, help patients use the incentive spirometer, etc. Occupational therapists are a resource for planning activities of daily living that patients need to modify or relearn. A mental health advanced practice nurse or psychologist should be used for severe anxiety.

A patient was given a patch test to determine what allergen was responsible for their atopic dermatitis. The provider prescribes a steroid cream. What important instructions should the nurse give to the patient? a. Apply the cream generously to affected areas. b. Apply a thin coat to affected areas, especially the face. c. Apply a thin coat to affected areas; avoid the face and groin. d. Apply an antihistamine along with applying a thin coat of steroid to affected areas.

ANS: C The patient should avoid the face and groin area as these areas are sensitive and may become irritated or excoriated. An antihistamine cream would also excoriate the area if the pruritus is cause by an allergen. There may be a need to administer oral steroid if the rash is generalized.

38. The patient has had a stroke that has affected the ability to speak. The patient becomes extremely frustrated when trying to speak. The patient responds correctly to questions and instructions but cannot form words coherently. Which type of aphasia is the patient experiencing? a. Sensory b. Receptive c. Expressive d. Combination

ANS: C The two types of aphasias are sensory (or receptive) and motor (or expressive). The patient cannot form words coherently, indicating expressive or motor aphasia is present. The patient responds correctly to questions and instructions, indicating receptive or sensory aphasia is not present. Patients sometimes suffer a combination of receptive and expressive aphasia, but this is not the case here.

15. On admission, a patient weighs 250 pounds. The weight is recorded as 256 pounds on the second inpatient day. Which condition will the nurse assess for in this patient? a. Anorexia b. Weight loss c. Fluid retention d. Increased nutritional intake

ANS: C This patient has gained 6 pounds in a 24-hour period. A weight gain of 5 pounds (2.3 kg) or more in a day indicates fluid retention problems, not nutritional intake. A weight loss is considered significant if the patient has lost more than 5% of body weight in a month or 10% in 6 months. A downward trend may indicate a reduction in nutritional reserves that may be caused by decreased intake such as anorexia.

36. The nurse is assessing skin turgor. Which technique will the nurse use? a. Press lightly on the forearm. b. Press lightly on the fingertips. c. Grasp a fold of skin on the sternal area. d. Grasp a fold of skin on the back of the hand.

ANS: C To assess skin turgor, grasp a fold of skin on the back of the forearm or sternal area with the fingertips and release. Since the skin on the back of the hand is normally loose and thin, turgor is not reliably assessed at that site. Pressing lightly on the forearm can be used to assess for pitting edema or pain or sense of touch. Pressing lightly on the fingertips and observing nail color is assessing capillary refill.

39. The nurse is assessing the tympanic membranes of an infant. Which action by the nurse demonstrates proper technique? a. Pulls the auricle upward and backward. b. Holds handle of the otoscope between the thumb and little finger. c. Uses an inverted otoscope grip while pulling the auricle downward and back. d. Places the handle of the otoscope between the thumb and index finger while pulling the auricle upward.

ANS: C Using the inverted otoscope grip while pulling the auricle downward and back is a common approach with infant/child examinations because it prevents accidental movement of the otoscope deeper into the ear canal, as could occur with an unexpected pediatric reaction to the ear examination. The other techniques could result in injury to the infant's tympanic membrane. Insert the scope while pulling the auricle upward and backward in the adult and older child. Hold the handle of the otoscope in the space between the thumb and index finger, supported on the middle finger.

35. The patient presents to the clinic with dysuria and hematuria. How does the nurse proceed to assess for kidney inflammation? a. Uses deep palpation posteriorly. b. Lightly palpates each abdominal quadrant. c. Percusses posteriorly the costovertebral angle at the scapular line. d. Inspects abdomen for abnormal movement or shadows using indirect lighting.

ANS: C With the patient sitting or standing erect, use direct or indirect percussion to assess for kidney inflammation. With the ulnar surface of the partially closed fist, percuss posteriorly the costovertebral angle at the scapular line. If the kidneys are inflamed, the patient feels tenderness during percussion. Use a systematic palpation approach for each quadrant of the abdomen to assess for muscular resistance, distention, abdominal tenderness, and superficial organs or masses. Light palpation would not detect kidney tenderness because the kidneys sit deep within the abdominal cavity. Posteriorly, the lower ribs and heavy back muscles protect the kidneys, so they cannot be palpated. Kidney inflammation will not cause abdominal movement. However, to inspect the abdomen for abnormal movement or shadows, the nurse should stand on the patient's right side and inspect from above the abdomen using direct light over the abdomen.

30. The nurse will anticipate inserting a Coudé catheter for which patient? a. An 8-year-old male undergoing anesthesia for a tonsillectomy b. A 24-year-old female who is going into labor c. A 56-year-old male admitted for bladder irrigation d. An 86-year-old female admitted for a urinary tract infection

ANS: C A Coudé catheter has a curved tip that is used for patients with enlarged prostates. This would be indicated for a middle-aged male who needs bladder irrigation. Coudé catheters are not indicated for children or women.

5. The patient is having lower abdominal surgery and the nurse inserts an indwelling catheter. What is the rationale for the nurse's action? a. The patient may void uncontrollably during the procedure. b. Local trauma sometimes promotes excessive urine incontinence. c. Anesthetics can decrease bladder contractility and cause urinary retention. d. The patient will not interrupt the procedure by asking to go to the bathroom.

ANS: C Anesthetic agents and other agents given during surgery can decrease bladder contractility and/or sensation of bladder fullness, causing urinary retention. Local trauma during lower abdominal and pelvic surgery sometimes obstructs urine flow, requiring temporary use of an indwelling urinary catheter. The patient is more likely to retain urine rather than experience uncontrollable voiding.

23. A nurse is caring for a patient who just underwent an intravenous pyelography that revealed a renal calculus obstructing the left ureter. What is the nurse's first priority in caring for this patient? a. Turn the patient on the right side to alleviate pressure on the left kidney. b. Encourage the patient to increase fluid intake to flush the obstruction. c. Monitor the patient for fever, rash, and difficulty breathing. d. Administer narcotic medications to the patient for pain.

ANS: C Assess for delayed hypersensitivity to the contrast media. Intravenous pyelography is performed by administering iodine-based dye to view functionality of the urinary system. Therefore, the first nursing priority is to assess the patient for an allergic reaction that could be life threatening. The nurse should then encourage the patient to drink fluids to flush dye resulting from the procedure. Narcotics can be administered but are not the first priority. Turning the patient on the side will not affect patient safety.

During the admission assessment, the nurse assesses the patient for fall risk. Which of the following has the greatest potential to increase the patient's risk for falls? a. The patient is 59 years of age. b. The patient walks 2 miles a day. c. The patient takes Benadryl (diphenhydramine) for allergies. d. The patient recently became widowed

ANS: C Benadryl (diphenhydramine) has the potential to cause drowsiness and dizziness as a side effect, thereby increasing the risk for falls. Over 60 is the age typically found on fall assessments that increase the risk for falls. Walking has many benefits, including increasing strength, which would be beneficial in decreasing risk.

15. The nurse suspects cystitis related to a lower urinary tract infection. Which clinical manifestation does the nurse expect the patient to report? a. Dysuria b. Flank pain c. Frequency d. Fever

ANS: C Cystitis is inflammation of the bladder; associated symptoms include hematuria, foul-smelling cloudy urine, and urgency/frequency. Dysuria is a common symptom of a lower urinary tract infection (bladder). Flank pain, fever, and chills are all signs of pyelonephritis (upper urinary tract).

2. A nurse is reviewing urinary laboratory results. Which finding will cause the nurse to follow up? a. Protein level of 2 mg/100 mL b. Urine output of 80 mL/hr c. Specific gravity of 1.036 d. pH of 6.4

ANS: C Dehydration, reduced renal blood flow, and increase in antidiuretic hormone secretion elevate specific gravity. Normal specific gravity is 1.0053 to 1.030. An output of 30 mL/hr or less for 2 or more hours would be cause for concern; 80 mL/hr is normal. The normal pH of urine is between 4.6 and 8.0. Protein up to 8 mg/100 mL is acceptable; however, values in excess of this could indicate renal disease.

38. Which observation by the nurse best indicates that a continuous bladder irrigation for a patient following genitourinary surgery is effective? a. Output that is smaller than the amount instilled b. Blood clots or sediment in the drainage bag c. Bright red urine turns pink in the tubing d. Bladder distention with tenderness

ANS: C If urine is bright red or has clots, increase irrigation rate until drainage appears pink, indicating successful irrigation. Expect more output than fluid instilled because of urine production. If output is smaller than the amount instilled, suspect that the tube may be clogged. The presence of blood clots indicates the patient is still bleeding, while sediment could mean an infection or bleeding. The bladder should not be distended or tender; the irrigant may not be flowing freely if these occur, or the tube may be kinked or blocked.

9. A nurse is caring for a male patient with urinary retention. Which action should the nurse take first? a. Limit fluid intake. b. Insert a urinary catheter. c. Assist to a standing position. d. Ask for a diuretic medication.

ANS: C In some patients just helping them to a normal position to void prompts voiding. A urinary catheter would relieve urinary retention, but it is not the first measure; other nursing interventions should be tried before catheterization. Reducing fluids would reduce the amount of urine produced but would not alleviate the urine retention and is usually not recommended unless the retention is severe. Diuretic medication would increase urine production and may worsen the discomfort caused by urine retention.

The emergency department has been notified of a potential bioterrorist attack. The nurse assigned to the department realizes that the most important task for safety in this situation is to a. Carry out the role and responsibilities of the nurse quickly and efficiently. b. Cluster all patients with the same symptoms to a specific part of the department. c. Determine the biologic agent and manage all patients using Standard Precautions. d. Prepare for post-traumatic stress associated with this bioterrorist attack

ANS: C It is essential to determine the agent and manage all patients who are symptomatic with the suspected or confirmed bioterrorism-related illness using Standard Precautions. For certain diseases, additional precautions may be necessary. Clustering patients may be helpful with staffing and, depending on the illness, may decrease the spread. All nurses every day should carry out their roles quickly and efficiently. Psychosocial concerns are important but are not the first priority at this moment.

The nurse preceptor recognizes the new nurse's ability to determine patient safety risks when which behavior is observed? a. Checking patient identification once every shift b. Multitasking by gathering two patients' medications c. Disposing of used needles in a red needle container d. Raising all four side rails per family request

ANS: C Needles, syringes, and other single-use injection devices should be used once and disposed of in safety red needle containers that will be disposed of properly. Patient identification should be checked multiple times a day, including before each medication, treatment, procedure, blood administration, and transfer, and at the beginning of each shift. Gathering more than one patient's medication increases the likelihood of error. Raising all four side rails is considered a restraint and requires special orders, assessment, and monitoring of the patient

28. A nurse is caring for an 8-year-old patient who is embarrassed about urinating in bed at night. Which intervention should the nurse suggest to reduce the frequency of this occurrence? a. "Set your alarm clock to wake you every 2 hours, so you can get up to void." b. "Line your bedding with plastic sheets to protect your mattress." c. "Drink your nightly glass of milk earlier in the evening." d. "Empty your bladder completely before going to bed."

ANS: C Nightly incontinence and nocturia are often resolved by limiting fluid intake 2 hours before bedtime. Setting the alarm clock to wake does not correct the physiological problem, nor does lining the bedding with plastic sheets. Emptying the bladder may help with early nighttime urination but will not affect urine produced throughout the night from late-night fluid intake.

The nurse enters the patient's room and notices a small fire in the headlight above the patient's bed. Immediately, the nurse assigns a nursing diagnosis of risk for injury with a goal for the patient to be safe. Which of the following actions should the nurse take first? a. Activate the alarm. b. Extinguish the fire. c. Remove the patient. d. Confine the fire.

ANS: C Nurses use the mnemonic RACE to set priorities in case of fire. All of these interventions are necessary, but this patient is in immediate danger with the fire being over his head and should be rescued and removed from the situation.

The nurse is discussing with a patient's physician the need for restraint. The nurse indicates that alternatives have been utilized. What behaviors would indicate that the alternatives are working? a. The patient continues to get up from the chair at the nurses' station. b. The patient apologizes for being "such a bother." c. The patient folds three washcloths over and over. d. The sitter leaves the patient alone to go to lunch.

ANS: C Offering diversionary activities such as something to hold is a way to keep the hands busy and provides an alternative to restraints. Assigning a room near the nurses' station or a chair at the desk can be an alternative for continuous monitoring. Getting up constantly can be cause for concern. Apologizing is not an alternative to restraints. Having a sitter sit with the patient to keep him occupied can be an alternative to restraints, but the sitter needs to be continuous.

31. A nurse is evaluating a nursing assistive personnel's (NAP) care for a patient with an indwelling catheter. Which action by the NAP will cause the nurse to intervene? a. Emptying the drainage bag when half full b. Kinking the catheter tubing to obtain a urine specimen c. Placing the drainage bag on the side rail of the patient's bed d. Securing the catheter tubing to the patient's thigh

ANS: C Placing the drainage bag on the side rail of the bed could allow the bag to be raised above the level of the bladder and urine to flow back into the bladder. The urine in the drainage bag is a medium for bacteria; allowing it to reenter the bladder can cause infection. A key intervention to prevent catheter-associated urinary tract infections is prevention of urine back flow from the tubing and bag into the bladder. All the rest are correct procedures and do not require follow-up. The drainage bag should be emptied when half full; an overfull drainage bag can create tension and pulling on the catheter, resulting in trauma to the urethra and/or urinary meatus and increasing risk for urinary tract infections. Urine specimens are obtained by temporarily kinking the tubing; a prolonged kink could lead to bladder distention. Failure to secure the catheter to the patient's thigh places the patient at risk for tissue injury from catheter dislodgment.

The nurse is presenting an educational session on safety for parents of adolescents. The nurse should include which of the following teaching points? a. Adolescents need unsupervised time with friends two to three times a week. b. Parents and friends should teach adolescents how to drive. c. Adolescents need information about the effects of beer on the liver. d. Adolescents need to be reminded to use seatbelts on long trips

ANS: C Providing information about drugs and alcohol is important because adolescents may choose to participate in risk-taking behaviors. Adolescents need to socialize but need supervision. Parents can encourage and support learning processes associated with driving, but organized classes can help to decrease motor vehicle accidents. Seatbelts should be used all the time.

16. Which assessment question should the nurse ask if stress incontinence is suspected? a. "Do you think your bladder feels distended?" b. "Do you empty your bladder completely when you void?" c. "Do you experience urine leakage when you cough or sneeze?" d. "Do your symptoms increase with consumption of alcohol or caffeine?"

ANS: C Stress incontinence can be related to intraabdominal pressure causing urine leakage, as would happen during coughing or sneezing. Asking the patient about the fullness of the bladder would rule out retention and overflow. An inability to void completely can refer to urge incontinence. Physiological causes and medications can effect elimination, but this is not related to stress incontinence.

The nurse knows that four categories of risk have been identified in the health care environment. Which of the following provides the best examples of those risks? a. Tile floors, cold food, scratchy linen, and noisy alarms b. Carpeted floors, ice machine empty, unlocked supply cabinet, and call light in reach c. Wet floors, pinching fingers in door, failure to use lift for patient, and alarms not functioning properly d. Dirty floors, hallways blocked, medication room locked, and alarms set

ANS: C The four categories are falls, patient-inherent accidents, procedure-related accidents, and equipment-related accidents. Wet floors contribute to falls, pinching finger in door is patient inherent, failure to use the lift is procedure related, and an alarm not functioning properly is equipment related. Tile floors and carpeted or dirty floors do not necessarily contribute to falls. Cold food, ice machine empty, and hallways blocked are not patient-inherent issues in the hospital setting but are more of patient satisfaction or infection control issues or fire safety issues. Scratchy linen, unlocked supply cabinet, and medication room locked are not procedure-related accidents. These are patient satisfaction issues and control of supply issues, and are examples of actually following a procedure correctly. Noisy alarms, call light within reach, and alarms set are not equipment-related accidents but are patient satisfaction issues and examples of following a procedure correctly.

A nurse is in the hallway assisting a patient to ambulate and hears an alarm sound. What is the best next step for the nurse to take? a. Seek out the source of the alarm. b. Wait to see if the alarm discontinues. c. Ask another nurse to check on the alarm. d. Continue ambulating the patient

ANS: C The nurse who heard the alarm has a duty to address it even though she is busy with another patient. Ask someone to check on the alarm. The nurse cannot leave the patient in the hallway to look for the source of the alarm and cause a potentially unsafe situation for this patient, but a patient on the unit may have an urgent need. Someone needs to seek out the source of the alarm and address it. Never ignore an alarm. Alarms are in place to maximize the safety of the patient. Waiting to see if an alarm stops may cause a delay in a possible emergency situation

The nurse is completing discharge education for the patient regarding home medications. Which patient behavior is an indication that the patient understands the directions regarding the antibiotic medication? a. The patient nods throughout the educational session. b. The patient reads the medication prescription out loud. c. The patient states, "I will finish the antibiotic in ten days." d. The patient asks where to get the prescription filled.

ANS: C The patient stating the time frame for when the medication will be complete is the best answer. Nodding, reading the prescription out loud, or knowing where to get the prescription filled does not indicate understanding regarding directions for taking the antibiotic.

10. Upon palpation, the nurse notices that the bladder is firm and distended; the patient expresses an urge to urinate. Which question is most appropriate? a. "Does your urinary problem interfere with any activities?" b. "Do you lose urine when you cough or sneeze?" c. "When was the last time you voided?" d. "Are you experiencing a fever or chills?"

ANS: C To obtain an accurate assessment, the nurse should first determine the source of the discomfort. Urinary retention causes the bladder to be firm and distended; time of last void is most appropriate. Further assessment to determine the pathology of the condition can be performed later. Questions concerning fever and chills, interference with any activities, and losing urine during coughing or sneezing focus on specific pathological conditions.

The nurse is caring for a patient who suddenly becomes confused and tries to remove an intravenous infusion. The nurse begins to develop a plan to care for the patient. Which nursing intervention should take priority? a. Gather restraint supplies. b. Try alternatives to restraint. c. Assess the patient. d. Call the physician for a restraint order

ANS: C When a patient becomes suddenly confused, the priority is to assess the patient, including checking laboratory test and oxygen status and treating and eliminating the cause of the change in mental status. If interventions and alternatives are exhausted, the nurse working with the physician may determine the need for restraints.

The nurse is caring for an 85-year-old woman 6 weeks following a hysterectomy secondary to ovarian cancer. The patient will need chemotherapy and irradiation on an outpatient basis. The nurse should identify and address which barriers to healing? (Select all that apply.) A. Can feed herself and prepare meals but cannot drive to the store B. Lives on a fixed income and can balance her checkbook C. Experiences stress incontinence D. Cannot participate in activities at the senior center E. Lives alone and has no nearby relatives F. Has no transportation to the oncology clinic

ANS: C, E, F The patient will not be able to get treatment if she has no transportation or no relatives that live nearby who can help her with recovery. Stress incontinence increases the risk of falls because of urgency and rushing to get to the bathroom. Income and social abilities are lower priorities during this phase of recovery.

2. Having misplaced a stethoscope, a nurse borrows a colleague's stethoscope. The nurse next enters the patient's room and identifies self, washes hands with soap, and states the purpose of the visit. The nurse performs proper identification of the patient before auscultating the patient's lungs. Which critical health assessment step should the nurse have performed? a.Running warm water over stethoscope b.Draping stethoscope around the neck c. Rubbing stethoscope with betadine d. Cleaning stethoscope with alcohol

ANS: D Bacteria and viruses can be transferred from patient to patient when a stethoscope that is not clean is used. The stethoscope should be cleaned before use on each patient with isopropyl alcohol. Running water over the stethoscope does not kill bacteria. Betadine is an inappropriate cleaning solution and may damage the equipment. Draping the stethoscope around the neck is not advised.

A nurse is providing range of motion to the shoulder and must perform external rotation. Which action will the nurse take? a.Moves patient's arm in a full circle b.Moves patient's arm cross the body as far as possible c.Moves patient's arm behind body, keeping elbow straight d.Moves patient's arm until thumb is upward and lateral to head with elbow flexed

ANS: D External rotation: With elbow flexed, move arm until thumb is upward and lateral to head. Circumduction: Move arm in full circle (Circumduction is combination of all movements of ball-and-socket joint.) Adduction: Lower arm sideways and across body as far as possible. Hyperextension: Move arm behind body, keeping elbow straight.

The nurse is assessing a patient's functional ability. Which activities most closely match the definition of functional ability? a. Healthy individual, works outside the home, uses a cane, well groomed b. Healthy individual, college educated, travels frequently, can balance a checkbook c. Healthy individual, works out, reads well, cooks and cleans house d. Healthy individual, volunteers at church, works part time, takes care of family and house

ANS: D Functional ability refers to the individual's ability to perform the normal daily activities required to meet basic needs; fulfill usual roles in the family, workplace, and community; and maintain health and well-being. The other options are good; however, each option has advanced or independent activities in the context of the option.

The nurse is admitting a patient who has been diagnosed as having had a stroke. The health care provider writes orders for "ROM as needed." What should the nurse do next? a.Restrict patient's mobility as much as possible. b.Realize the patient is unable to move extremities. c.Move all the patient's extremities. d.Further assess the patient.

ANS: D Further assessment of the patient is needed to determine what the patient is able to perform. Some patients are able to move some joints actively, whereas the nurse passively moves others. With a weak patient, the nurse may have to support an extremity while the patient performs the movement. In general, exercises need to be as active as health and mobility allow.

13. The nurse is observing the way a patient walks. Which aspect is the nurse assessing? a.Activity tolerance b.Body alignment c.Range of motion d.Gait

ANS: D Gait describes a particular manner or style of walking. Activity tolerance is the type and amount of exercise or work that a person is able to perform. Body alignment refers to the position of the joints, tendons, ligaments, and muscles while standing, sitting, and lying. Range of motion is the maximum amount of movement available at a joint in one of the three planes of the body: sagittal, frontal, or transverse.

A nurse is performing passive range of motion (ROM) and splinting on an at-risk patient. Which finding will indicate goal achievement for the nurse's action? a.Prevention of atelectasis b.Prevention of renal calculi c.Prevention of pressure ulcers d.Prevention of joint contractures

ANS: D Goal achievement for passive ROM is prevention of joint contractures. Contractures develop in joints not moved periodically through their full ROM. ROM exercises reduce the risk of contractures. Researchers noted that prompt use of splinting with prescribed ROM exercises reduced contractures and improved active range of joint motion in affected lower extremities. Deep breathing and coughing and using an incentive spirometer will help prevent atelectasis. Adequate hydration helps prevent renal calculi and urinary tract infections. Interventions aimed at prevention of pressure ulcers include positioning, skin care, and the use of therapeutic devices to relieve pressure.

10. During a routine pediatric history and physical, the parents report that their child was a very small, premature infant that had to stay in the neonatal intensive care unit longer than usual. They state that the infant was yellow when born and developed an infection that required "every antibiotic under the sun" to reach a cure. Which exam is a priority for the nurse to conduct on the child? a. Cardiac b. Respiratory c. Ophthalmic d. Hearing acuity

ANS: D Hearing is the priority. Risk factors for hearing problems include low birth weight, nonbacterial intrauterine infection, and excessively high bilirubin levels. Hearing loss due to ototoxicity (injury to auditory nerves) can result from high maintenance doses of antibiotics. Cardiac, respiratory, and eye examinations are important assessments but are not relevant to this child's condition.

A nurse is caring for an immobile patient. Which metabolic alteration will the nurse monitor for in this patient? a.Increased appetite b.Increased diarrhea c.Increased metabolic rate d.Altered nutrient metabolism

ANS: D Immobility disrupts normal metabolic functioning: decreasing the metabolic rate, altering the metabolism of carbohydrates, fats, and proteins; causing fluid, electrolyte, and calcium imbalances; and causing gastrointestinal disturbances such as decreased appetite and slowing of peristalsis, leading to constipation.

13. The nurse is urgently called to the gymnasium regarding an injured student. The student is crying in severe pain with a malformed fractured lower leg. Which proper sequence will the nurse follow to perform the initial assessment? a. Light palpation, deep palpation, and inspection b. Inspection, light palpation, and deep palpation c. Auscultation and light palpation d. Inspection and light palpation

ANS: D Inspection is the use of vision and hearing to distinguish normal from abnormal findings. Light palpation determines areas of tenderness and skin temperature, moisture, and texture. Deep palpation is used to examine the condition of organs, such as those in the abdomen. Caution is the rule with deep palpation. Deep palpation is performed after light palpation; however, deep palpation is not performed on a fractured leg. Auscultation is used to evaluate sound and is not used to assess a fractured leg.

A nurse is preparing a care plan for a patient who is immobile. Which psychosocial aspect will the nurse consider? a.Loss of bone mass b.Loss of strength c.Loss of weight d.Loss of hope

ANS: D Loss of hope is a psychosocial aspect. Patients with restricted mobility may have some depression. Depression is an affective disorder characterized by exaggerated feelings of sadness, melancholy, dejection, worthlessness, emptiness, and hopelessness out of proportion to reality. All the rest are physiological aspects: bone mass, strength, and weight.

7. A school nurse recognizes a belt buckle-shaped ecchymosis on a 7-year-old student. When privately asked about how the injury occurred, the student described falling on the playground. Which action will the nurse take next? a. Talk to the principal about how to proceed. b. Disregard the finding based upon child's response. c. Interview the patient in the presence of the teacher. d. Contact social services and report suspected abuse.

ANS: D Most states mandate a report to a social service center if nurses suspect abuse or neglect. When abuse is suspected, the nurse interviews the patient in private, not with a teacher. Observe the behavior of the individual for any signs of frustration, explanations that do not fit his or her physical presentation, or signs of injury. The nurse knows how to proceed and does not need to talk to the principal about what to do. Disregarding the finding is not advised because victims often will not complain or report that they are in an abusive situation.

A nurse is providing passive range of motion (ROM) for a patient with impaired mobility. Which technique will the nurse use for each movement? a.Each movement is repeated 5 times by the patient. b.Each movement is performed until the patient experiences pain. c.Each movement is completed quickly and smoothly by the nurse. d.Each movement is moved just to the point of resistance by the nurse.

ANS: D Passive ROM exercises are performed by the nurse. Carry out movements slowly and smoothly, just to the point of resistance; ROM should not cause pain. Never force a joint beyond its capacity. Each movement needs to be repeated 5 times during the session. The patient moves all joints through ROM unassisted in active ROM.

A patient asks the nurse what the purpose of the Wood's light is. Which response by the nurse is accurate? a. "We will put an anesthetic on your skin to prevent pain." b. "The lamp can help detect skin cancers." c. "Some patients feel a pressure-like sensation." d. "It is used to identify the presence of infectious organisms and proteins associated with specific skin conditions."

ANS: D The Wood's light examination is the use of a black light and darkened room to assist with physical examination of the skin. The examination does not cause discomfort.

The patient has the nursing diagnosis of Impaired physical mobility related to pain in the left shoulder. Which priority action will the nurse take? a.Encourage the patient to do self-care. b.keep the patient as mobile as possible. c.Encourage the patient to perform ROM. d.Assist the patient with comfort measures.

ANS: D The diagnosis related to pain requires the nurse to assist the patient with comfort measures so that the patient is then willing and more able to move. Pain must be controlled so the patient will not be reluctant to initiate movement. The diagnosis related to reluctance to initiate movement requires interventions aimed at keeping the patient as mobile as possible and encouraging the patient to perform self-care and ROM.

The nurse is developing an interdisciplinary plan of care using the Roper-Logan-Tierney Model of Nursing for a patient who is currently unconscious. Which interventions would be most critical to developing a plan of care for this patient? a. Eating and drinking, personal cleansing and dressing, working and playing b. Toileting, transferring, dressing, and bathing activities c. Sleeping, expressing sexuality, socializing with peers d. Maintaining a safe environment, breathing, maintaining temperature

ANS: D The most critical aspects of care for an unconscious patient are safe environment, breathing, and temperature. Eating and drinking are contraindicated in unconscious patients. Toileting, transferring, dressing, and bathing activities are BADLs. Sleeping, expressing sexuality, and socializing with peers are a part of the Roper-Logan-Tierney Model of Nursing; however, these are not the most critical for developing the plan of care in an unconscious patient.

33. A patient in the emergency department is reporting left lower abdominal pain. Which proper order will the nurse follow to perform the comprehensive abdominal examination? a. Percussion, palpation, auscultation b. Percussion, auscultation, palpation c. Inspection, palpation, auscultation d. Inspection, auscultation, palpation

ANS: D The order of an abdominal examination differs slightly from that of other assessments. Begin with inspection and follow with auscultation. By using auscultation before palpation, the chance of altering the frequency and character of bowel sounds is lessened.

The nurse is instructing the nursing assistant to prevent pressure ulcers in a frail older patient. Which action indicates the nursing assistant has understood the nurse's teaching? a. Bathing and drying the skin vigorously to stimulate circulation b. Keeping the head of the bed elevated 30 degrees c. Limiting intake of fluid and offer frequent snacks d. Turning the patient at least every 2 hours

ANS: D The patient should be turned at least every 2 hours as permanent damage can occur in 2 hours or less. If skin assessment reveals a stage I ulcer while on a 2-hour turning schedule, the patient must be turned more frequently. Limiting fluids will prevent healing; however, offering snacks is indicated to increase healing particularly if they are protein based, because protein plays a role in healing. Use of doughnuts, elevated heads of beds, and overstimulation of skin may all stimulate, if not actually encourage, dermal decline.

The nurse is caring for an older-adult patient with a diagnosis of urinary tract infection (UTI). Upon assessment the nurse finds the patient confused and agitated. How will the nurse interpret these assessment findings? a.These are normal signs of aging. b.These are early signs of dementia. c.These are purely psychological in origin. d.These are common manifestation with UTIs.

ANS: D The primary symptom of compromised older patients with an acute urinary tract infection or fever is confusion. Acute confusion in older adults is not normal; a thorough nursing assessment is the priority. With the diagnosis of urinary tract infection, these are not early signs of dementia and they are not purely psychological.

The patient is unable to move self and needs to be pulled up in bed. What will the nurse do to make this procedure safe? a.Place the pillow under the patient's head and shoulders. b.Do by self if the bed is in the flat position. c.Place the side rails in the up position. d.Use a friction-reducing device.

ANS: D This is not a one-person task. Helping a patient move up in bed without help from other co-workers or without the aid of an assistive device (e.g., friction-reducing pad) is not recommended and is not considered safe for the patient or the nurse. Remove the pillow from under head and shoulders and place it at the head of the bed to prevent striking the patient's head against the head of the bed. When pulling a patient up in bed, the bed should be flat to gain gravity assistance, and the side rails should be down.

5. A head and neck physical examination is completed on a 50-year-old female patient. All physical findings are normal except for fine brittle hair. Which laboratory test will the nurse expect to be ordered, based upon the physical findings? a.Oxygen saturation b. Liver function test c. Carbon monoxide d. Thyroid-stimulating hormone test

ANS: D Thyroid disease can make hair thin and brittle. Liver function testing is indicated for a patient who has jaundice. Oxygen saturation will be used for cyanosis. Cherry-colored lips indicate carbon monoxide poisoning.

20. Upon assessment, the patient is breathing normally and has normal vesicular lung sounds. Which expected inspiratory-to-expiratory breath sounds will the nurse hear? a. The expiration phase is longer than the inspiration phase. b. The inspiratory phase lasts exactly as long as the expiratory phase. c. The expiration phase is 2 times longer than the inspiration phase. d. The inspiratory phase is 3 times longer than the expiratory phase.

ANS: D Vesicular breath sounds are normal breath sounds; the inspiratory phase is 3 times longer than the expiratory phase. Bronchovesicular breath sounds have an inspiratory phase equal to the expiratory phase. Bronchial breath sounds have an expiration phase longer than the inspiration phase at a 3:2 ratio.

To help decrease the threat of a melanoma in a blonde-haired, fair-skinned patient at risk, the nurse would advise the patient to do which of the following? a. Apply sunscreen 1 hour prior to exposure. b. Drink plenty of water to prevent hot skin. c. Use vitamins to help prevent sunburn by replacing lost nutrients. d. Apply sunscreen 30 minutes prior to exposure.

ANS: D Wearing sunglasses and sunscreen are recommended by the National Cancer Institute. Drinking water will help with heat exhaustion but will not prevent melanoma. Green tea, fish oil, soy products, and vitamin E are thought to be helpful in minimizing the risk of developing melanoma; however, vitamins do not prevent burn.

The nurse is caring for a patient with a urinary catheter. After the nurse empties the collection bag and disposes of the urine, the next step is to a. Use alcohol-based gel on hands. b. Wash hands with soap and water. c. Remove eye protection and dispose of in garbage. d. Remove gloves and dispose of in garbage.

ANS: D After disposing of the urine, the first step in removing personal protective equipment is removing gloves and disposing of them properly. In this scenario, the next step would be to remove eye protection followed by hand hygiene. Wash hands if the hands are visibly soiled; otherwise the use of alcohol-based gel is indicated for routine decontamination of hands.

A home health nurse is performing a home assessment for safety. Which of the following comments by the patient would indicate a need for further education? a. "I will schedule an appointment with a chimney inspector next week." b. "Daylight savings is the time to change batteries on the carbon monoxide detector." c. "If I feel dizzy when using the heater, I need to have it inspected." d. "When it is cold outside in the winter, I can warm my car up in the garage."

ANS: D Allowing a car to run in the garage introduces carbon monoxide into the environment and decreases the available oxygen for human consumption. Garages should be opened and not just cracked to allow fresh air into the space and allay this concern. Checking the chimney and heater, changing the batteries on the detector, and following up on symptoms such as dizziness, nausea, and fatigue are all statements that would indicate that the individual has understood the education.

19. A nurse is reviewing results from a urine specimen. What will the nurse expect to see in a patient with a urinary tract infection? a. Casts b. Protein c. Crystals d. Bacteria

ANS: D Bacteria in the urine along with other symptoms support a diagnosis of urinary tract infection. Crystals would be seen with renal stone formation. Casts indicate renal disease. Protein indicates kidney function and damage to the glomerular membrane such as in glomerulonephritis.

21. Which clinical manifestation will the nurse expect to observe in a patient with excessive white blood cells present in the urine? a. Reduced urine specific gravity b. Increased blood pressure c. Abnormal blood sugar d. Fever with chills

ANS: D Fever and chills may be observed. The presence of white blood cells in urine indicates a urinary tract infection or inflammation. Overhydration, early renal disease, and inadequate antidiuretic hormone secretion reduce specific gravity. Increased blood pressure is associated with renal disease or damage and some medications. Abnormal blood sugars would be seen in someone with ketones in the urine or a patient with diabetes.

13. A nurse is inserting a catheter into a female patient. When the nurse inserts the catheter, no urine is obtained. The nurse suspects the catheter is not in the urethra. What should the nurse do? a. Throw the catheter way and begin again. b. Fill the balloon with the recommended sterile water. c. Remove the catheter, wipe with alcohol, and reinsert after lubrication. d. Leave the catheter in the vagina as a landmark for insertion of a new, sterile catheter.

ANS: D If no urine appears, the catheter may be in the vagina. If misplaced, leave the catheter in the vagina as a landmark to indicate where not to insert, and insert another sterile catheter. The catheter should be left in place until the new, sterile catheter is inserted. The balloon should not be filled since the catheter is in the vagina. The catheter must be sterile; using alcohol will not make the catheter sterile.

The nurse knows that children in late infancy and toddlerhood are at risk for injury owing to a. Learning to walk. b. Trying to pull up on furniture. c. Being dropped by a caregiver. d. Growing ability to explore and oral activity.

ANS: D Injury is a leading cause of death in children over age 1, which is closely related to normal growth and development because of the child's increased oral activity and growing ability to explore the environment.

The nurse discussed threats to adult safety with a college group. Which of the following statements would indicate understanding of the topic? a. "Our campus is safe; we leave our dorms unlocked all the time." b. "As long as I have only two drinks, I can still be the designated driver." c. "I am young, so I can work nights and go to school with 2 hours' sleep." d. "I guess smoking even at parties is not good for my body."

ANS: D Lifestyle choices frequently affect adult safety. Smoking conveys great risk for pulmonary and cardiovascular disease. It is prudent to secure belongings. When an individual has been determined to be the designated driver, that individual does not consume alcohol, beer, or wine. Sleep is important no matter the age of the individual and is important for rest and integration of learning. The average young adult needs 6 1/2 to 8 hours of sleep each night.

The nurse is precepting a student nurse and is careful to check with the student all components of the medication process. The nurse explains to the student that most errors occur in a. Ordering and transcribing. b. Dispensing and administering. c. Dispensing and transcribing. d. Ordering and administering

ANS: D Most medication errors occur in the ordering and administering stages of the medication process

A patient has been admitted and placed on fall precautions. The nurse explains to the patient that interventions for the precautions include a. Encouraging visitors in the early evening. b. Placing all four side rails in the "up" position. c. Checking on the patient once a shift. d. Placing a high risk for falls armband on the patient.

ANS: D Placing a high risk for falls armband on the patient encourages communication among the whole interdisciplinary team. Anyone who interacts with the patient should see this armband, understand its meaning, and assist the patient as necessary. The timing of visitors would not affect falls. All four side rails are considered a restraint and can contribute to falling. Individuals on high risk for fall alerts should be checked frequently, at least every hour

The nurse is providing information regarding safety and accidental poisoning to a grandmother who will be taking custody of a 1-year-old grandchild. Which of the following comments would indicate that the grandmother needs further instruction? a. "The number for poison control is 800-222-1222." b. "Never induce vomiting if my grandchild drinks bleach." c. "I should call 911 if my grandchild loses consciousness." d. "If my grandchild eats a plant, I should provide syrup of ipecac."

ANS: D Syrup of ipecac to induce vomiting after ingestion of a poison has not been proven effective in preventing poisoning. This medication should not be administered to the child. The poison control number is 800-222-1222. After a caustic substance such as bleach has been drunk, do not induce vomiting. This can cause further burning and injury as the medication is eliminated. Loss of consciousness associated with poisoning requires calling 911.

26. A nurse is watching a nursing assistive personnel (NAP) perform a postvoid bladder scan on a female with a previous hysterectomy. Which action will require the nurse to follow up? a. Palpates the patient's symphysis pubis b. Wipes scanner head with alcohol pad c. Applies a generous amount of gel d. Sets the scanner to female

ANS: D The nurse will follow up if the NAP sets the scanner to female. Women who have had a hysterectomy should be designated as male. All the rest are correct and require no follow-up. The NAP should palpate the symphysis pubis, the scanner head should be cleaned with an alcohol pad, and a generous amount of gel should be applied.

The older patient presents to the emergency department after stepping in front of a car at a crosswalk. After the patient has been triaged, the nurse interviews the patient. Which of the following comments would require follow-up by the nurse? a. "I try to exercise, so I walk that block almost every day." b. "I waited and stepped out when the traffic sign said go." c. "The car was going too fast, the speed limit is 20." d. "I was so surprised; I didn't see or hear the car coming."

ANS: D The patient did not see or hear the car coming. As patients age, sensory impairment can increase the risk for injury. This statement by the patient would require follow-up by the nurse. The patient needs hearing and eye examinations. Exercise is important at every stage of development. The patient seemed to comprehend how to cross an intersection correctly and was able to determine the speed of the car.

18. To obtain a clean-voided urine specimen from a female patient, what should the nurse teach the patient to do? a. Cleanse the urethral meatus from the area of most contamination to least. b. Initiate the first part of the urine stream directly into the collection cup. c. Drink fluids 5 minutes before collecting the urine specimen. d. Hold the labia apart while voiding into the specimen cup.

ANS: D The patient should hold the labia apart to reduce bacterial levels in the specimen. The urethral meatus should be cleansed from the area of least contamination to greatest contamination (or front to back). The initial stream flushes out microorganisms in the urethra and prevents bacterial transmission in the specimen. Drink fluids 30 minutes before giving a specimen.

The patient is confused, is trying to get out of bed, and is pulling at the intravenous infusion tubing. These data would help to support a nursing diagnosis of a. Risk for poisoning. b. Knowledge deficit. c. Impaired home maintenance. d. Risk for injury.

ANS: D The patient's behaviors support the nursing diagnosis of risk for injury. The patient is confused, is pulling at the intravenous line, and is trying to climb out of bed. Injury could result if the patient falls out of bed or begins to bleed from a pulled line. Nothing in the scenario indicates that this patient lacks knowledge or is at risk for poisoning. Nothing in the scenario refers to the patient's home maintenance

17. The patient has a catheter that must be irrigated. The nurse is using a needleless closed irrigation technique. In which order will the nurse perform the steps, starting with the first one? 1. Clean injection port. 2. Inject prescribed solution. 3. Twist needleless syringe into port. 4. Remove clamp and allow to drain. 5. Clamp catheter just below specimen port. 6. Draw up prescribed amount of sterile solution ordered. a. 3, 2, 6, 1, 5, 4 b. 5, 6, 1, 2, 3, 4 c. 1, 5, 6, 3, 2, 4 d. 6, 5, 1, 3, 2, 4

ANS: D The steps for irrigating with a needleless closed irrigation technique is as follows: Draw up in a syringe the prescribed amount of medication or sterile solution; clamp indwelling retention catheter just below specimen port; using circular motion, clean injection port with antiseptic swab; insert tip of needleless syringe using twisting motion into irrigation port; slowly and evenly inject fluid into catheter and bladder; and withdraw syringe, remove clamp, and allow solution to drain into drainage bag.

29. A nurse is inserting an indwelling urinary catheter for a male patient. Which action will the nurse take? a. Hold the shaft of the penis at a 60-degree angle. b. Hold the shaft of the penis with the dominant hand. c. Cleanse the meatus 3 times with the same cotton ball from clean to dirty. d. Cleanse the meatus with circular strokes beginning at the meatus and working outward.

ANS: D Using the uncontaminated dominant hand, cleanse the meatus with cotton balls/swab sticks, using circular strokes, beginning at the meatus and working outward in a spiral motion. Repeat 3 times using a clean cotton ball/swabstick each time. With the nondominant hand (now contaminated), retract the foreskin (if uncircumcised) and gently grasp the penis at the shaft just below the glans. Hold the shaft of the penis at a right angle to the body.

A nurse is performing a mental status examination and asks an adult patient what the statement "Don't cry over spilled milk" means. Which area is the nurse assessing?

Abstract thinking

A nurse is caring for a group of patients. Which patient will the nurse see first?

An adult with an S4 heart sound

A teen female patient reports intermittent abdominal pain for 12 hours. No dysuria is present. Which action will the nurse take when performing an abdominal assessment?

Ask the patient about the color of her stools.

An advanced practice nurse is preparing to assess the external genitalia of a 25-year-old American woman of Chinese descent. Which action will the nurse do first?

Assess the patient's feelings about the examination.

The home health nurse is teaching a patient and family about hand hygiene in the home. Which situation will cause the nurse to emphasize washing hands before and after? a.Shaking hands b.Performing treatments c.Opening the refrigerator d.Working on a computer

B

The infection control nurse is reviewing data for the medical-surgical unit. The nurse notices an increase in postoperative infections from Aspergillus. Which type of health care-associated infection will the nurse report? a.Vector b.Exogenous c.Endogenous d.Suprainfection

B

The nurse is admitting a patient with an infectious disease process. Which question will be most appropriate for a nurse to ask about the patient's susceptibility to this infectious process? a."Do you have a spouse?" b."Do you have a chronic disease?" c."Do you have any children living in the home?" d."Do you have any religious beliefs that will influence your care?"

B

The nurse is caring for a patient who has a bloodborne pathogen. The nurse splashes blood above the glove to intact skin while discontinuing an intravenous (IV) infusion. Which step(s) will the nurse take next? a.Obtain an alcohol swab, remove the blood with an alcohol swab, and continue care. b.Immediately wash the site with soap and running water, and seek guidance from the manager. c.Do nothing; accidentally getting splashed with blood happens frequently and is part of the job. d.Delay washing of the site until the nurse is finished providing care to the patient.

B

The nurse is changing linens for a postoperative patient and feels a prick in the left hand. A nonactivated safe needle is noted in the linens. For which condition is the nurse most at risk? a.Diphtheria b.Hepatitis B c.Clostridium difficile d.Methicillin-resistant Staphylococcus aureus

B

The nurse is providing an education session to an adult community group about the effects of smoking on infection. Which information is most important for the nurse to include in the educational session? a.Smoke from tobacco products clings to your clothing and hair. b.Smoking affects the cilia lining the upper airways in the lungs. c.Smoking can affect the color of the patient's fingernails. d.Smoking tobacco products can be very expensive.

B

The patient experienced a surgical procedure, and Betadine was utilized as the surgical prep. Two days postoperatively, the nurse's assessment indicates that the incision is red and has a small amount of purulent drainage. The patient reports tenderness at the incision site. The patient's temperature is 100.5° F, and the WBC is 10,500/mm3. Which action should the nurse take first? a.Plan to change the surgical dressing during the shift. b.Utilize SBAR to notify the primary health care provider. c.Reevaluate the temperature and white blood cell count in 4 hours. d.Check to see what solution was used for skin preparation in surgery.

B

The patient has contracted a urinary tract infection (UTI) while in the hospital. Which action will most likely increase the risk of a patient contracting a UTI? a.Reusing the patient's graduated receptacle to empty the drainage bag. b.Allowing the drainage bag port to touch the graduated receptacle. c.Emptying the urinary drainage bag at least once a shift. d.Irrigating the catheter infrequently.

B

Which nursing action will most likely increase a patient's risk for developing a health care-associated infection? a.Uses surgical aseptic technique to suction an airway b.Uses a clean technique for inserting a urinary catheter c.Uses a cleaning stroke from the urinary meatus toward the rectum d.Uses a sterile bottled solution more than once within a 24-hour period

B

Which process will be required after exposure of a nurse to blood by a cut from a used scalpel in the operative area? a.Placing the scalpel in a needle safe container b.Testing the patient and offering treatment to the nurse c.Removing sterile gloves and disposing of in kick bucket d.Providing a medical evaluation of the nurse to the manager

B


Set pelajaran terkait

I have a Dream" by Martin Luther King, Jr. from Rosa Parks: My Story by Rosa Parks with Jim Haskins "There Is a Longing . .." by Chief Dan George "I Hear America Singing" by Walt Whitman

View Set

TExES EC-6 Fine arts, health and physical education, Fine Arts, Health, and PE

View Set

Public speaking chapters 11,12,13,17

View Set

World History Cumulative Exam *Review*

View Set

Cat 9 - KBUR (Class C) to KFUL (Class D)

View Set

Chapter 5: States of Consciousness

View Set

Unit 6 - Imperial China Collapses

View Set